You are on page 1of 264

FACULTAD DE INGENIERIA CIVIL Y ARQUITECTURA

ESCUELA ACDEMICA PROFESIONAL DE INGENIERA CIVIL

SOLUCIONARIO DEL EXAMEN PARCIAL (CICLO 1.2010 (25/08/10) )


Ing. Civil Clifton Paucar y Montenegro - REG. CIP. 45773
ABASTECIMIENTOS DE AGUA Y ALCANTARILLADO - CIC404

RESP.DEL REA HIDRAULICA :


ASIGNATURA:

RPTA A

i)

Iniciamos el proceso de estimacin calculando la dotacin promedio CATASTRAL, en funcin al consumo promedio cuanti
35,000

Poblacin total localidad BETA.


ZONA A :

Densidad poblacional promedio :

DATOS

TIPO1( unid.)=
Lit./viv./da
900
700
500

Mximo
Mnimo
Promedio

TOTAL VIVIENDAS
Consumo promedio por edificacin/ da =

Poblacin en el Ao 2009:

4.00
TIPO2( unid.) =
Lit./viv./da
950
725
520

500
Lit./hab./da
230
170
150
1,330

500 x 500 400 x 520 250 x 550 180 x 600


( 500 400 250 180 )

15.00%

35,000

Densidad poblacional promedio por edificacin


Dotacin para el AO 2009 en base al consumo por tipo de edificacin
=
(x C.R:Includo 17% de prdidas y desperdicios en la red de reciente instalacin)
Dotacin para el AO 2009 en base al consumo por habitante
=
(xC.C:Incidencia futura de incremento de consumo por la tendencia de crecimiento comercial)
TOMANDO EL VALOR MAYOR DE DOTACIN TENDREMOS
Consumo promedio instantaneo para la zona A
ZONA B :
DATOS

7.00

Densidad poblacional promedio :


TIPO1( unid.)=

Lit./viv./da
1,200
1,000
950

Mximo
Mnimo
Promedio

200

Lit./hab./da
140
120
100

TOTAL VIVIENDAS
Consumo promedio por edificacin/ da =
Poblacin en el Ao 2009:

TIPO2( unid.) =

Lit./viv./da
1,180
1,020
948
2,420

50.00%

2, 00 x 950 500 x 948 800 x 888 920 x 1000


( 2, 00 500 800 920 )
35,000

Densidad poblacional promedio por edificacin


Dotacin para el AO 2009 en base al consumo por tipo de edificacin
=
(x C.R:Includo 45% de prdidas y desperdicios en la red de instalacin antigua)
Dotacin para el AO 2009 en base al consumo por habitante
=
(xC.C:Incidencia futura de incremento de consumo al mantener la tendencia de crecimiento sostenido de tipo residencial)
TOMANDO EL VALOR MAYOR DE DOTACIN TENDREMOS
Consumo promedio instantaneo para la zona B
ZONA C :
DATOS

Mximo
Mnimo
Promedio

Densidad poblacional promedio :


TIPO1( unid.)=

Lit./viv./da
500
400
350

4.00
800

Lit./hab./da
120
100
80

TIPO2( unid.) =

Lit./viv./da
500
450
400

FACULTAD DE INGENIERIA CIVIL Y ARQUITECTURA


ESCUELA ACDEMICA PROFESIONAL DE INGENIERA CIVIL

SOLUCIONARIO DEL EXAMEN PARCIAL(CICLO 1.2010 (25/08/10) )

Ing. Civil Clifton Paucar y Montenegro - REG. CIP. 45773


ABASTECIMIENTOS DE AGUA Y ALCANTARILLADO - CIC404

RESP.DEL REA HIDRAULICA :


ASIGNATURA

800 x 350 750 x 400 620 x 466 880 x 500


( 800 750 620 880 )

Consumo promedio por edificacin/ da =

Poblacin en el Ao 2009:

35.00%

35,000

Densidad poblacional promedio por edificacin


Dotacin para el AO 2009 en base al consumo por tipo de edificacin
=
(x C.R:Includo 17% de prdidas y desperdicios en la red de reciente instalacin)
Dotacin para el AO 2009 en base al consumo por habitante
=
(xC.C:Incidencia futura de incremento de consumo por la tendencia de crecimiento comercial)
TOMANDO EL VALOR MAYOR DE DOTACIN TENDREMOS
Consumo promedio instantaneo para la zona C

Rpta. A y B

Densidad Promedio Poblacional de las Zonas A,B(c/sistema) y C(s/sistema).


ZONA CON SERVICIO DE AGUA

CUADRO DE RESPUESTAS A Y B.DEMANDA CATASTRAL PROMEDIO:


Densidad prom.poblac.(hab./edif.)
Poblacin Total ao 2009 ( habitantes )
Dotacin para el ao 2009 (Lt./hab./da)
Consumo prom.ao 2009 ( Lt./seg. )
Q de diseo del sistema ao 2010(Dato)

Rpta. C

Caudal de Diseo para las Zonas A,B y C.


CUADRO DE RESPUESTA PARA EL INCISO C.-

DEMANDA CATASTRAL POR EDIFICACIN Includo/PRDIDAS:


Densidad prom.poblac.(hab./edif.)
Poblacin Total ao 2009 ( habitantes )
Dotacin para el ao 2009 calculada por edificacin sin prdidas(Lt./hab./da)
Dotacin para el ao 2009 Inc. Prdidas segn datos suministrados 80%-75%-39% (Lt./hab./da)
Dotacin para el ao 2015 Inc. Prdidas segn datos proyectados 80%-75%-39% (Lt./hab./da)
Dotacin para el ao 2021Inc. Prdidas segn datos proyectados 80%-75%-39% (Lt./hab./da)
Dotacin para el ao 2031 Inc. Prdidas segn datos proyectados 60%-45%-39% (Lt./hab./da)
Consumo prom.ao 2009 sin / Prdidas (Lt./seg. )
Consumo prom.ao 2009 includo/ Prdidas (Lt./seg. )
Poblacin proyectada ao 2015
Consumo prom.ao 2015 sin / Prdidas (Lt./seg. )
Consumo prom.ao 2015 Includo / Prdidas (Lt./seg. )
Poblacin proyectada ao 2021
Consumo prom.ao 2021 sin / Prdidas (Lt./seg. )
Consumo prom.ao 2021 Includo / Prdidas (Lt./seg. )
Poblacin proyectada ao 2031
Consumo prom.ao 2031 sin / Prdidas (Lt./seg. )

Consumo prom.ao 2031 Includo / Prdidas (Lt./seg. )


Q de diseo del sistema ao 2010(Dato)

EL ANLISIS DE LA VARIACIN DE CONSUMO EN EL TIEMPO SE EFECTUAR CON LA DECISIN DE USAR


CON LOS DATOS (ZONAS URBANAS-DONDE EL INCREMENTO DEL CONSUMO NO SOLO DEPENDE DEL C
RURALES-DONDE LA DOTACIN SE BASA EN EL USO DOMESTICO Y SU VARIACIN DEPENDE DEL IN
QPROMEDIO DE DISEO SEGN LAS CARACTERISTICAS DE CONSUMO DIARIO Y HORARIO, LOS FACTORES

ii)

Efectuamos las estimaciones de las demanda por variaciones de consumo diarias en funcin a los datos sumin
de determinar los valores de K2.

Para determinar el caudal medio en Lt./seg. durante el ao 2009, se proceder a desarrollar el cuadro que contie
representa en forma analitica a la curva masa (Produccin y Demanda vrs. Tiempo), teniendo como base que se consu
que es de 551 Lt.seg. con el cul se diseo el reservorio en el ao 2005, proyecto que entr en funcionamiento el ao
el ao 2020.
DATO:

Qdisp. Mx.=
GASTO
HORAS

0
2
4
6
8
10
12
14
16
18
20
22
24

PROM.
@ 2 horas
% del Qprom.

1,551

Litros/seg.

CUADRO 02.GASTO
PROM @ 2HR.

VOLUMEN

(54.45 Lt./seg.)

(54.45 Lt./seg.)

(Lt./seg.)

(Lts./s.)

(Lts.)

25.00%
30.00%
40.00%
100.00%
160.00%
110.00%
160.00%
115.00%
90.00%
140.00%
150.00%
80.00%
25.00%

13.613
16.335
21.780
54.450
87.120
59.895
87.120
62.618
49.005
76.230
81.675
43.560
13.613
667.01

107,813
137,214
274,428
509,652
529,254
529,254
539,057
401,843
450,846
568,458
450,846
205,823
4,704,488

(Lts./s.)

(Lts.)

POR DA

54.45
VALOR SIN APLICACIN DEL FACTOR DE MAYORACION K1 =1.3

Como se tiene como dato que la persistencia de consumo de los valores mximos diarios es de 30 a 33% se utilizar el cuadro No. 01 para estimar el valo
de las curvas masa en ambos casos con fines de comparacin de los consumos.

FACULTAD DE INGENIERIA CIVIL Y ARQUITECTU

ESCUELA ACDEMICA PROFESIONAL DE INGENIERA C

SOLUCIONARIO DEL EXAMEN PARCIAL(CICLO 1.2010(26/08


RESP.DEL REA HIDRAULICA :

Ing. Civil Clifton Paucar y Montenegro - REG. CIP. 45773

ASIGNATURA

ABASTECIMIENTOS DE AGUA Y ALCANTARILLADO - CIC404

50,000,000

50,000,000

45,000,000
40,000,000
35,000,000
30,000,000

25,000,000
20,000,000
15,000,000
10,000,000

5,000,000
0
0

SIMULACIN PRODUCC.(2009) (54.45 Lt./seg.) (Lts.)


SIMULACIN PRODUCC.(2009) (54.45 Lt./seg.) (Lts.)

VOLUM. ACUMUL. Cons.da mx. (Lts.)

HORAS

GASTO
PROM.
@ 2 horas
% del Qprom.
(Lt./seg.)

0
2
4
6
8
10
12
14
16
18
20
22
24

30.00%
35.00%
45.00%
105.00%
165.00%
120.00%
200.00%
115.00%
95.00%
145.00%
160.00%
85.00%
30.00%

LOCALIDAD (Consumo del da mximo

CUADRO 01.GASTO
PROM @ 2HR.
(54.45Lt./seg.)
(Lts./s.)

POR DA

VOLUMEN
(54.45 Lt./seg.)
(Lts.)

16.34
19.06
24.50
57.17
89.84
65.34
108.90
62.62
51.73
78.95
87.12
46.28
16.34

127,440
156,816
294,012
529,236
558,648
627,264
617,472
411,660
470,448
597,852
480,240
225,432

724.19

5,096,520

(Lts./s.)

(Lts.)
58.99

Rpta. D:

k1=

1.30

(Reglam.)

FACULTAD DE INGENIERIA CIVIL Y ARQUITECTURA


ESCUELA ACDEMICA PROFESIONAL DE INGENIERA CIVIL

SOLUCIONARIO DEL EXAMEN PARCIAL(CICLO 1.2010 26/08/10) )


RESP.DEL REA HIDRAULICA :

Ing. Civil Clifton Paucar y Montenegro - REG. CIP. 45773

ASIGNATURA

ABASTECIMIENTOS DE AGUA Y ALCANTARILLADO - CIC404

CUADRO DE PRODUCCIN Y DEFICIT EN EL SERVICIO DE AGUA EN EL AO BASE DE OBSERVACION


PRODUCCIN Y DEFICIT DEL SERVICIO EN EL AO 2009

PRODUCCIN

DATO
Caudal que se deriva a la zona A

Caudal que se deriva a la zona B


Caudal que se deriva a la zona C
DEMANDA CATASTRAL CALCULADA
SUPERAVIT O DEFICIT EN EL AO 2009

Rpta. E:

iii)

Efectuamos las estimaciones del crecimiento poblacional en funcin a la ecuacin caracteristicas de crecimiento poblacion
porcentuales.

Y i 2 5 .3 3 e

0 .0 0 0 0 9 X i

ZONA A :
CUADRO DE PROYECCION DE POBLACIN ZONA A
AO

2004
2009
2014
2019
2024
2029
2034
2039
2044
2049
2054
2059
2064
2069
2074

%
CREC.POB.

4,490
5,250
6,079
6,970
7,913
8,896
9,908
10,937
11,972
13,004
14,026
15,031
16,015
16,975
17,908

16.909735
15.791779
14.656435
13.527024
12.426353
11.374221
10.384034
9.465555
8.623659
7.858765
7.168163
6.548261
5.993284
5.497203

Habitantes con servicio en el 2009

kg(2009-2004)
kg(2014-2009)
kg(2019-2014)

POBLACIN

TASAS CREC.
POB.

152
166
178
189
197
202
206
207
206
204
201
197
192
187
5,250

0.01564
0.01466
0.01368

POB.ESTIM. CUANDO ENTRE EN OPERACIN EL PROYECTO:

Pob.A(2013)

5,914

Hab.

FACULTAD DE INGENIERIA CIVIL Y ARQUITECTURA


ESCUELA ACDEMICA PROFESIONAL DE INGENIERA CIVIL

SOLUCIONARIO DEL EXAMEN PARCIAL(CICLO 1.2010 (26/08/10) )


RESP.DEL REA HIDRAULICA :

Ing. Civil Clifton Paucar y Montenegro - REG. CIP. 45773

ABASTECIMIENTOS DE AGUA Y ALCANTARILLADO - CIC404

ASIGNATURA

ZONA B :

Y i 7 5 .5 5 0 .0 0 0 3 8 X

CUADRO DE PROYECCION DE POBLACIN ZONA B


AO

POBLACIN

CREC.POB.

2004
2009
2014
2019
2024
2029
2034
2039
2044

TASAS CREC.
POB.

13,702
17,500
31,885
59,837
118,650
261,786
719,987
3,233,786
45,414,925

80.45676
82.20000
87.66630
98.28806
120.63700
175.02868
349.14506
1,304.38868

760
2,877
5,590
11,763
28,627
91,640
502,760
8,436,228

POB.ESTIM. CUANDO ENTRE EN OPERACIN EL PROYECTO :


Pob.B(2013)

ZONA C :

29,008

Hab.

Yi 44.88 e 0.00002

Xi

CUADRO DE PROYECCION DE POBLACIN ZONA C


AO

POBLACIN

CREC.POB.

2004
2009
2014
2019
2024
2029
2034
2039
2044
kg(2009-2004)
kg(2014-2009)
kg(2019-2014)

7,767
12,250
18,041
24,807
31,904
38,776
45,121
50,849
55,982

38.422847
47.276379
37.501119
28.609270
21.538688
16.362151
12.694525
10.095106

Habitantes con servicio en el 2009

TASAS CREC.
POB.

897
1,158
1,353
1,419
1,374
1,269
1,146
1,027
0

0.09113
0.07742
0.06370

POB.ESTIM. CUANDO ENTRE EN OPERACIN EL PROYECTO :


Pob.C(2013)

16,882

Hab.

FACULTAD DE INGENIERIA CIVIL Y ARQUITECTURA


ESCUELA ACDEMICA PROFESIONAL DE INGENIERA CIVIL

SOLUCIONARIO DEL EXAMEN PARCIAL(CICLO 1.2010 (26/08/10) )


RESP.DEL REA HIDRAULICA :
ASIGNATURA

Rpta.F:

Ing. Civil Clifton Paucar y Montenegro - REG. CIP. 45773


ABASTECIMIENTOS DE AGUA Y ALCANTARILLADO - CIC404

SE DESARROLLA EL SGTE. CUADRO PARA DETERMINAR EL AO EN QUE EL SISTEMA EXISTENTE CUBRIRA LA DEMANDA TOTAL DE A, B Y C.

CUADRO DE POBLACIN Y CONSUMOS PROMEDIOS PROYECTADOS CON DEMANDA CONSTANTE(C


DEMANDA CATASTRAL:
Poblacin Total ao 2009 ( habitantes )
Dotacin para el ao 2009(Lt./hab./da)
Consumo promedio ao 2009 ( Lt./seg. )
Poblacin Total ao 2013( habitantes )
Consumo promedio ao 2013( Lt./seg. )
Poblacin Total ao 2015 ( habitantes )
Consumo promedio ao 2015 ( Lt./seg. )
Poblacin Total ao 2021 ( habitantes )
Consumo promedio ao 2021 ( Lt./seg. )
Poblacin Total ao 2031 ( habitantes )
Consumo promedio ao 2031 ( Lt./seg. )
Poblacin Total ao 2034 ( habitantes )
Consumo promedio ao 2034( Lt./seg. )

Usando la Expresin de CAPEN (Pg.7 AA-Simn Arocha) adaptada al dato base de cada zona
poblacional:
G = k p 0.125

Adaptando los valores de k para cada zona poblacional tendremos:


Zona A:
122.43
Con stos valores elaboramos el cuadro No. 8 corregido

CUADRO DE POBLACIN Y CONSUMOS PROMEDIOS PROYECTADOS CON DEMANDA VARIABLE


DEMANDA CATASTRAL:
Poblacin Total ao 2009 ( habitantes )
Consumo promedio ao 2009 ( Lt./seg. )
Poblacin Total ao 2013 ( habitantes )
Consumo promedio ao 2013 ( Lt./seg. )
Poblacin Total ao 2015 ( habitantes )
Consumo promedio ao 2015( Lt./seg. )
Poblacin Total ao 2021 ( habitantes )
Consumo promedio ao 2021 ( Lt./seg. )
Poblacin Total ao 2031 ( habitantes )
Consumo promedio ao 2031 ( Lt./seg. )
Poblacin Total ao 2034 ( habitantes )
Consumo promedio ao 2034 ( Lt./seg. )

CUADRO DE POBLACIN Y CONSUMOS PROMEDIOS PROYECTADOS CON DEMANDA VARIABLE


DEMANDA CATASTRAL:
Poblacin Total ao 1999 ( habitantes )
Dotacin para el ao 1999(Lt./hab./da)
Consumo promedio ao 1999 ( Lt./seg. )
Poblacin Total ao 2005 ( habitantes )
AO DE
OBSOLENCIA DEL Dotacin para el ao 2005(Lt./hab./da)
PROY.ANT.

AO DE
OBSOLENCIA DEL
PROY.ANT.
AO DE
BASE DE FORM.

INICIO DE
OPERACIN

Consumo promedio ao 2005 ( Lt./seg. )


Poblacin Total ao 2009 ( habitantes )
Dotacin para el ao 2009(Lt./hab./da)
Consumo promedio ao 2009 ( Lt./seg. )
Poblacin Total ao 2012 ( habitantes )

Dotacin para el ao 2012(Lt./hab./da)


Consumo promedio ao 2012 ( Lt./seg. )
Poblacin Total ao 2015 ( habitantes )
Dotacin para el ao 2015(Lt./hab./da)
Consumo promedio ao 2015( Lt./seg. )
1ERA. DECADA Poblacin Total ao 2019 ( habitantes )
DESDE EL AO Dotacin para el ao 2019(Lt./hab./da)
BASE DE OBSERV. Consumo promedio ao 2019( Lt./seg. )
Poblacin Total ao 2021 ( habitantes )

FINAL DE
OPERACIN 15
AOS

Dotacin para el ao 2021(Lt./hab./da)


Consumo promedio ao 2021 ( Lt./seg. )
Poblacin Total ao 2026 ( habitantes )
Dotacin para el ao 2026(Lt./hab./da)
Consumo promedio ao 2026 ( Lt./seg. )
Poblacin Total ao 2029 ( habitantes )
Dotacin para el ao 2029 (Lt./hab./da)
Consumo promedio ao 2029 ( Lt./seg. )
Poblacin Total ao 2031 ( habitantes )
Dotacin para el ao 2031(Lt./hab./da)
Consumo promedio ao 2031 ( Lt./seg. )
Poblacin Total ao 2036 ( habitantes )
Dotacin para el ao 2036 (Lt./hab./da)
Consumo promedio ao 2036 ( Lt./seg. )
Poblacin Total ao 2039 ( habitantes )
Dotacin para el ao 2039(Lt./hab./da)
Consumo promedio ao 2039 ( Lt./seg. )
Poblacin Total ao 2081 ( habitantes )
Dotacin para el ao 2081(Lt./hab./da)
Consumo promedio ao 2081 ( Lt./seg. )

Rpta. G:

Hasta el MES DE MARZO DEL ao 2029 para las zonas A, B y C, con el uso de la Represa + Prdidas + Variac. de consumos

FACULTAD DE INGENIERIA CIVIL Y ARQUITECTU

ESCUELA ACDEMICA PROFESIONAL DE INGENIERA C

EXAMEN PARCIAL(CICLO 1.2010(26/08/10) )


SOLUCIONARIO DEL EXAMEN PARCIAL(CICLO 1.2010(26/08
ABASTECIMIENTOS DE AGUA Y ALCANTARILLADO - CIC404

ASIGNATURA

Para determinar el periodo de diseo de las obras complementarias usaremos los resultados de
necesidades de agua de las zonas "A", "B" y "C" segn la demanda y las ecuaciones del crecimiento p

iv)

SE CALCULA EL PERODO DE DISEO EN BASE A LOS DATOS CON QUE SE CUENTA, QUE INDICA QUE SON OBRAS DE AMPLIACIN PARA LAS ZONAS
MAYOR.

USANDO LOS DOS TERMINOS DE LA FORMULA DE LAURIA:

X1 * = X 1 + X 2 = [( 2.6 ( 1 - a) ^ 1.12) / (r) ] + [ ( 0.3 (1 - a) Xo ^ 0.85 ) / (r)^ 1/2 ]


Dato:

r= 0.2

CONSIDERANDO LA SIGUIENTE DESCRIPCIN SIMPLIFICADA:

N..C. = CAPTACIN NUEVA


N.L.C. = NUEVA LNEA DE CONDUCCIN
N.P.T.= NUEVA PLANTA DE TRATAMIENTO
N.R.P.=NUEVO RESERVORIO PRINCIPAL

ESTRUCT.

C. EJEC.

C.OP.yMANT.

COMPLEM.

( S/.)

( S/.)

76,184,955

10,665,894

103,611,539
83,803,451
76,184,955
12,189,593
2,724,435
1,871,891,384
23,042,429
3,036,285
161,215,898
9,338,263
2,423,223,187

11,846,057
23,617,336
10,665,894
2,285,549
381,421
262,064,794
3,225,940
759,071
35,354,364
2,334,566
363,200,885

01

N.C.

02
03
04
05
06
07
08
09
10
11

N.L.C.
N.P.T.
N.R.P.
N.C.D.
N.R.Z."A
N.R.Z."B
N.R.Z."C"
AMP.R "A"
R.N."B"
R.N."C"

LOS COSTOS DE LAS OBRAS COMPLEMENTARIAS LO ESTIMAMOS EN BASE AL

Tasa activa :

20 %

FACULTAD DE INGENIERIA CIVIL Y ARQUITECTU

ESCUELA ACDEMICA PROFESIONAL DE INGENIERA C

EXAMEN PARCIAL(CICLO 1.2010 (26/08/10) )


SOLUCIONARIO DEL EXAMEN PARCIAL(CICLO 1.2010 (26/08
ABASTECIMIENTOS DE AGUA Y ALCANTARILLADO - CIC404

ASIGNATURA

INICIAMOS EL CALCULO ASUMIENDO UN PERIODO DE DISEO DE 18 AOS ( 2029-2012), QUE ES EL DETERMINADO POR LA
A,B Y C, DE ACUERDO A SU CRECIMIENTO POBLACIONAL CORREGIDO POR EL MODELO LINEAL DE LOS DATOS SUMINISTRA

1.-RELACIN ENTRE CAPTACIN Y NUEVA CAPTACIN PARALELA (N.C.)


AO
Q (m3/seg.)
Cap. Reserv(m3)
COST.EJEC.
2,008
0.151
0
COST.MANT..

2,008

0.151

2.-RELACIN ENTRE LINEA DE CONDUCCIN Y LA NUEVA LNEA DE CONDUCCIN PARALELO(N.L.C.)


COST.EJEC.

2,008

0.151

COST.MANT..

2,008

0.151

3.-RELACIN ENTRE PLANTA DE TRATAMIENTO Y LA NUEVA PLANTA DE TRATAMIENTO PARALELO(N.P.T.)


COST.EJEC.
COST.MANT..

2,008
2,008

0.151
0.151

0
0

4.-RELACIN ENTRE RESERVORIO PRINCIPAL Y NUEVO RESERVORIO PRINCIPAL PARALELO(N.R.P.)


COST.EJEC.
2,008
0.151
0
COST.MANT..
2,008
0.151
0
5.-RELACIN ENTRE CAJA DE DISTRIBUCIN Y NUEVA CAJA DE DISTRIBUCIN EN PARALELO
COST.EJEC.
2,008
0.151
0
COST.MANT..
2,008
0.151
0
6.-RELACIN ENTRE RESERVORIO PRINCIPAL Y NUEVO RESERVORIO ZONAL EN A (N.R.Z.A.)
COST.EJEC.
2,008
0.151
0
COST.MANT..
2,008
0.151
0
7.-RELACIN ENTRE RESERVORIO PRINCIPAL Y NUEVO RESERVORIO ZONAL EN B (N.R.Z.B.)
COST.EJEC.
2,008
0.151
0
COST.MANT..
2,008
0.151
0
8.-RELACIN ENTRE RESERVORIO PRINCIPAL Y NUEVO RESERVORIO ZONAL EN C (N.R.Z.C.)
COST.EJEC.
2,008
0.151
0
COST.MANT..
2,008
0.151
0
9.-REDES EN A (R.D.N.A.)
COST.EJEC.
2,008
COST.MANT..
2,008

0.151
0.151

..
..

2,008
2,008

0.151
0.151

..
..

2,008
2,008

0.151
0.151

..
..

10.-REDES EN B (R.D.N.B.)

COST.EJEC.
COST.MANT..
11.-REDES EN C (R.D.N.C.)

COST.EJEC.
COST.MANT..

ESTIMAREMOS EL VALOR DE Xo PARA CADA ZONA SEGN EL CRECIMIENTO TEORICO ANUAL DE LA POBLACIN, TENIENDO
Demanda en Deficit ( a2012 - a2009) / Crecimiento cte.de la demanda
Xo =

Se considera el ao 2012, como aquel en donde empezar a operar el proyecto(Estudio y Ejecucin = 2 aos: 2010 y

ZONA A:
2012

Pob.ref.(2026)=
Pob.ref.(2012)=

9,300
5,914

hab.
hab.

2005
Pob.ref.(2005)=
5,402
hab.
DEFICIT periodo:
aos
DEFICIT Poblacin:
hab.
2012-2005=
7
16,442-15,652
512
DEFICIT Consum.en el periodo(Lit./da):
CRECIMIENTO CTE.DEMANDA DE DISEO
Dotacin cte. Asumida
del ao 2012 para el
Dotacin para el
periodo (2005-2012)
180.76
602.52
ao 2012
en Lt./hab./da
Lt./hab./da
DOTACION(2029)
Demanda(2012)
Lt.//da

ZONA B:

Pob.ref.(2026)=

Demanda cte. (20052012) Lt./da


445,066 hab.

2012

Pob.ref.(2012)=

29,008 hab.

92,547.1

44,070

2005
Pob.ref.(2005)=
18,262 hab.
DEFICIT periodo:
aos
DEFICIT Poblacin:
hab.
2012-2005=
7
48,565-40,111 =
10,746
CRECIMIENTO CTE.DEMANDA DE DISEO
DEFICIT Consum.en el periodo(Lit./da):
Dotacin cte. Asumida
el ao 2010para el
Dotacin para el
periodo de periodo
157.49
251.10
ao
(2005-2012) en
2012Lt./hab./da
Lt./hab./da
DOTACION(2029)
Demanda(2012)
Demanda cte. (20051,692,439.12
385,475
Lt.//da
2011) Lt./da
ZONA C: Pob.ref.(2026)=
41,314 hab.
2012

Pob.ref.(2012)=

1990
Pob.ref.(1990)=
DEFICIT periodo:
aos
2012-1990=
22
DEFICIT Consum.en el periodo(Lit./da):
Dotacin para el
ao 2012
Lt./hab./da

112.65

Demanda(2012)
Lt.//da

1,901,833.27

TOTALES

No. ORDEN

01
02

0 hab.
DEFICIT Poblacin:
hab.
22,207-0 =
16,882
CRECIMIENTO CTE.DEMANDA DE DISEO
Dotacin cte. Asumida
el ao 2010 para el
periodo de periodo
167.64
(1990-2012) en
Lt./hab./da
DOTACION(2029)
Demanda cte. (2000128,641
2012) Lt./da

Pob.2026

495,680 hab.

Pob.2012
Pob.2009

51,804 hab.
35,000 hab.

ESTRUCT.
COMPLEM.

N.C.
N.L.C.

16,882 hab.

Xo

X2 (S D)

7.00
7.00

3.02
3.11

03

N.P.T.

7.00

2.52

04
05
06
07
08
09
10
11

N.R.P.

7.00
7.00
2.00
4.00
15.00
2.00
4.00
15.00

3.02
2.85
1.04
1.87
5.76
0.91
1.70
5.03
30.82

N.C.D.
N.R.Z."A"
N.R.Z."B"
N.R.Z."C"
AMPLIAC.RD."A"
RD.N."B"
RD.N."C"

FACULTAD DE INGENIERIA CIVIL Y ARQUITECTU

ESCUELA ACDEMICA PROFESIONAL DE INGENIERA C

SOLUCIONARIO DEL EXAMEN PARCIAL(CICLO 1.2010 (26/08


RESP.DEL REA HIDRAULICA :
ASIGNATURA

Ing. Civil Clifton Paucar y Montenegro - REG. CIP. 45773


ABASTECIMIENTOS DE AGUA Y ALCANTARILLADO - CIC404

Los volumenes de consumo del Reservorio principal y de los reservorios zonales "A", "B" y "C" se cal
RPTA H.El volumen inicialmente construdo ha sido calculado con el Q disponible de 151 l/s. con los datos del cuadro 02 de con
entre los mximos y promedios de 10 a 13% en la parte inicial, cuando se determinaron los coeficientes de mayoracin.

ZONA
A

Coeficiente de variacin de consumo entre


CONSIDERANDO CON LA CONSTRUCCIN DE LAS O.C LA PROD
el 2008 y el 2026
3.4000
Del Cuadro 2 sumamos los valores absolutos mximos multiplicado po

1.5730

1.4688

Coef.CPROM.

2.1470

MENOS EL VOL. RES. EFIC. AL 2008 =


DIF. PARA DISEO R. PRINC.COMP.2026

Resultados teoricos:
CONJUNTO DE RESERVORIOS ZONALES
Para Ao (2026) =

Para el reservorio zonal "B" que ser abastecido por el Reservorio Prin
al no tener datos del consumo horario se optara por la recomendacin
25% ( 230.75 x 24 x 60 x 60)/1000 =

ZONA A
Solo para diferencia de Q(2025-1990)=
ZONA B

Para Ao (2026) =

-18.70
25% ( 277.59 x 24 x 60 x 60) =
0.00

Para Ao (2026) =

ZONA C

25% ( 91.43 x 24 x 60 x 60) =


0.00

El predimensionamiento de las tuberas a partir de la caja de distribucin se tomaran del cuadro No. 09
del cuadro No.03, as como el cuadro de predimensionamiento de tuberas con dimetros y velocidade
Se deber verificar dos valores para tomar de ellos el mayor:

RPTA I.-

i ) ( k1 x Qprom ) + Qincendio
ii )

K2 x Qprom.

DEL REGLAMENTO SE OBTIENE EL VALOR DE k1=1.3 AL NO EXISTIR DATOS DE VARIACIONES DIARIAS DEL A

DEL CUADRO 03 SE OBTIENE EL VALOR DE K2 para el da mximo anual = 3.4


USAMOS K 2 = 3.4 AL NO INDICARNOS LAS MEDIDAS DE CONTINGENICA PARA USAR EL VALOR DE Q MAX.
CONSIDERANDO QUE LOS VALORES MXIMOS DEL CUADRO 03 SE PRESENTAN AL AO CON MENOS DE 30
USANDO LOS DATOS DE CAUDALES MEDIOS DEL CUADRO 9 PARA EL AO 2040 SE TIENE:

ZONA "A"
i)
ii )

( 1.3 x 107.31 ) + 50 =
3.4 x 107.31
=

189.50 Lit./seg.
364.82 Lit./seg.

ZONA "B"

i)
ii )

( 1.3 X 858.86 ) + 50 =
3.4 x 858.86
=

1,166.52 Lit/seg.
2, 920.12 Lit./seg.

ZONA "C"
i ) ( 1.3 X 118,051.37 ) + 50 =

153,516.78 Lit/seg.

ii )

401,374.66 Lit./seg.

3.4 x 118,051.37

PULG.

SE UTILIZAN LOS DIAMETRO MAYORES ( NORMA.FAB. ITINTEC)

MM.

SE UTILIZAN LOS DIAMETRO MAYORES ( NORMA.FAB. ISO)

Montenegro - REG. CIP. 45773


AGUA Y ALCANTARILLADO - CIC404

HOJA N 01/09

ATASTRAL, en funcin al consumo promedio cuantificado por tipo de EDIFICACIN Y HABITANTE, as como el consumo promedio en Lt./seg.
Habitantes
(RED EXISTENTE)

habitantes por vivienda

400
Lit./hab./da
210
180
125

TIPO3( unid.) =
Lit./viv./da
1,000
750
550

250
Lit./hab./da
222
188
166

520 250 x 550 180 x 600


400 250 180 )

TIPO4( unid.) =
Lit./viv./da
1,250
780
600
=

Hab.

528.95

5,250

hab.

4.00

hab

132.24

Lt/hab./da

150.63

Lt/hab./da

180
Lit./hab./da
206
199
188
Lit./edif./da

Se toma el mayor.

=
habitantes por vivienda
500

Lit./hab./da
200
150
130

Lt./seg.

(RED EN PROYECCIN)
800
TIPO4( unid.) =

TIPO3( unid.) =

Lit./viv./da
1,150
1,050
888

948 800 x 888 920 x 1000


500 800 920 )
Hab.

9.15

Lit./hab./da
190
175
150

920

Lit./viv./da
1,240
1,090
1,000
=

17,500 hab.

7.00 hab

Lit./hab./da
210
195
168

948.10 Lit./edif./da

135.44 Lt/hab./da
148.58 Lt/hab./da

sostenido de tipo residencial)


=
habitantes por vivienda
750

Lit./hab./da
190
150
135

TIPO3( unid.) =

Lit./viv./da
500
480
466

30.09 Lt./seg.
(RED EN PROYECCIN)
620
TIPO4( unid.) =

Lit./hab./da
150
125
112

Lit./viv./da
580
550
500

880

Lit./hab./da
110
90
85

Montenegro - REG. CIP. 45773


AGUA Y ALCANTARILLADO - CIC404

HOJA N 02/09

400 620 x 466 880 x 500


750 620 880 )

Hab.

429.15 Lit./edif./da

12,250 hab.

4.00 hab
107.29 Lt/hab./da
101.47 Lt/hab./da

15.21 Lt./seg.

onas A,B(c/sistema) y C(s/sistema).


ZONA SIN SERVICIO DE AGUA

Zona "A"

Zona "B"

Zona "C"

Total

DEFICIT./SUPERAV.

4.00

7.00

4.00

5,250
150.63
9.15

17,500
148.58
30.09

12,250
107.29
15.21

35,000.00

Zona "A"

Zona "B"

Zona "C"

Total

4.00

7.00

4.00

5,250

17,500

12,250

150.63

148.58

107.29

271.13

260.02

149.13

271.13

260.02

149.13

271.13

260.02

149.13

271.13

260.02

149.13

9.15

30.09

15.21

54.45

16.48

52.67

21.14

90.29

6,246.00

34,762.00

19,198.00

60,206.00

10.89

59.78

23.84

94.51

19.60

104.61

33.14

157.35

7,348.00

83,363.00

27,645.00

118,356.00

12.81

143.36

34.33

190.50

23.06

250.88

47.72

321.66

0.00

0.00

0.00

0.00

0.00

0.00

0.00

0.00

54.45

911.9%

551.00

35,000.00

165.8%

166.5%

168.9%

0.00

0.00

0.00

9.15

0.00

#DIV/0!

PROM.:

#DIV/0!

SE EFECTUAR CON LA DECISIN DE USAR PARA EL CALCULO DE LA DOTACIN INCREMENTADA , UNA ECUACIN LINEAL
O DEL CONSUMO NO SOLO DEPENDE DEL CRECIMIENTO POBLACIONAL) O EL MODELO MATEMATICO DE CAPEN (ZONAS
MESTICO Y SU VARIACIN DEPENDE DEL INCREMENTO POBLACIONAL), PARA LO CUL ANTES DEBEMOS ESTIMAR EL
ONSUMO DIARIO Y HORARIO, LOS FACTORES DE MAYORACIN K1 Y K2 Y LA POBLACIN FUTURA.

consumo diarias en funcin a los datos suministrados del diario promedio horario y del diario maximo horario en Lt./seg., a fin

ceder a desarrollar el cuadro que contiene las variaciones horarias del consumo promedio durante el ao, que
Tiempo), teniendo como base que se consume 9.15 Lt./seg. (2009), y que es la produccin total de la captacin (100%)
yecto que entr en funcionamiento el ao 2006 con un Pd=15 aos, debiendo cumplir con eficiencia la demanda hasta

Caudal aprovechable(dato).

LOCALIDAD ( Consumo promedio ao 2009 )


VOLUM.(2009)

PRODUCC.(2009)

VOLUM. DE

SIMULACIN

VOLUM. DE

ACUM.CONSUMO

ACUM.24Hr.

CONS.RESERV.

PRODUCC.(2009)

CONS.RESERV.

(54.45 Lt./seg.)

(9.5 Lt./seg.)

(Demanda 54.45)

(54.45 Lt./seg.)

(Demanda 186.30 Lt./s)

(Lts.)

(Lts.)

(Lts.)

(Lts.)

(Lts.)

107,813
245,027
519,455
1,029,107
1,558,361
2,087,615
2,626,672
3,028,515
3,479,361
4,047,819
4,498,665
4,704,488
4,704,488
4,704.49

3,967,200
7,934,400
11,901,600
15,868,800
19,836,000
23,803,200
27,770,400
31,737,600
35,704,800
39,672,000
43,639,200
47,606,400
47,606,400

3,859,387
7,689,373
11,382,145
14,839,693
18,277,639
21,715,585
25,143,728
28,709,085
32,225,439
35,624,181
39,140,535
42,901,912

392,040
784,080
1,176,120
1,568,160
1,960,200
2,352,240
2,744,280
3,136,320
3,528,360
3,920,400
4,312,440
4,704,480
4,704,480

284,227
539,053
656,665
539,053
401,839
264,625
117,608
107,805
48,999
-127,419
-186,225
-8

(M3)
k2prom.09:

Reserv.9.5 l/s.=

39,042,525

Vol.diseo inic.reservorio 95:

Reserv(54.45l/s.)=

842,890

Vol.diseo ampliac.reservor 2008

1.60
39,042.5

M3

842.9

a 33% se utilizar el cuadro No. 01 para estimar el valor de K2 y el Volumen de consumo del reservorio principal de ampliacin. Se muestra los graficos

DE INGENIERIA CIVIL Y ARQUITECTURA

ELA ACDEMICA PROFESIONAL DE INGENIERA CIVIL

RIO DEL EXAMEN PARCIAL(CICLO 1.2010(26/08/10) )

Montenegro - REG. CIP. 45773


AGUA Y ALCANTARILLADO - CIC404

HOJA N 03/09

10

12

14

16

18

20

22

PRODUCC.(2009) ACUM.24Hr. (9.5 Lt./seg.) (Lts.)


PRODUCC.(2009) ACUM.24Hr. (54.45Lt./seg.) (Lts.)

LOCALIDAD (Consumo del da mximo ao 2009 )


VOLUM.
ACUMUL.
Cons.da mx.
(Lts.)

PRODUCC.(2009)
ACUM.24Hr.
(54.45Lt./seg.)
(Lts.)

VOLUM. DE
CONSUM.RES. (100%)
Cons.da mx.
(Lts.)

127,440
284,256
578,268
1,107,504
1,666,152
2,293,416
2,910,888
3,322,548
3,792,996
4,390,848
4,871,088
5,096,520
5,096,520

3,967,200
7,934,400
11,901,600
15,868,800
19,836,000
23,803,200
27,770,400
31,737,600
35,704,800
39,672,000
43,639,200
47,606,400
47,606,400

3,839,760
7,650,144
11,323,332
14,761,296
18,169,848
21,509,784
24,859,512
28,415,052
31,911,804
35,281,152
38,768,112
42,509,880
42,509,880

5,096.52

(M3)

Reserv(54.45l./s.)=
38,670,120
Vol.diseo ampliac.reservor 2008 por cons.mx.horar.

k2dis.(2008):

2.00

(Coef.da de mx.consumo)

k2dis.09:

1.60

(Consumo Promedio)

38,670.00

M3

Montenegro - REG. CIP. 45773


AGUA Y ALCANTARILLADO - CIC404

HOJA N 04/09

EL AO BASE DE OBSERVACION
Producc.

Habitantes

Habitantes

=
=

=
=
=

(Lt./seg.)

con servicio

sin servicio

551.00
9.15
9.15
1.66%
30.09
15.21

5,250
5,250
100.00%
0
0

0
0
0.00%
17,500
12,250

54.45
911.94%

5,250
17.65%

29,750
100.00%
5,250

A:

TOTAL HABITANTES:

B+C:

29,750

a ecuacin caracteristicas de crecimiento poblacional suministrados para cada zona, procesados utilizando el mtodo de incrementos

Montenegro - REG. CIP. 45773

CURVA DE CRECIMIENTO POBLACIONAL ZONA B

20,000

18,000

16,000

POBLACIN

14,000

12,000

10,000

8,000

6,000

4,000

2,000

0
2004

2009

2014

2019

2024

2029

2034

2039

2044

AO

POBLACIN

TASAS CREC. POB.

2049

2054

2059

2064

2069

2074

AGUA Y ALCANTARILLADO - CIC404

HOJA N 05/09

CURVA DE CRECIMIENTO POBLACIONAL ZONA A


50,000,000
45,000,000
40,000,000

POBLACIN

35,000,000
30,000,000
25,000,000
20,000,000
15,000,000
10,000,000
5,000,000
0
2004

2009

2014

2019

2024

2029

2034

2039

2044

2029

2034

2039

2044

AO

POBLACIN

TASAS CREC. POB.

CURVA DE CRECIMIENTO POBLACIONAL ZONA A


60,000

50,000

POBLACIN

40,000

30,000

20,000

10,000

0
2004

Montenegro - REG. CIP. 45773


AGUA Y ALCANTARILLADO - CIC404

EXISTENTE CUBRIRA LA DEMANDA TOTAL DE A, B Y C.

2009

2014

2019

2024

AO

POBLACIN

TASAS CREC. POB.

HOJA N 06/09

YECTADOS CON DEMANDA CONSTANTE(Considerando que la demanda es cte. durante el periodo de diseo)
Zona "A"
5,250
150.63
9.15
5,914
10.31
6,246
10.89
7,348
12.81
9,300
16.21
9,908

Zona "B"
17,500
148.58
30.09
29,008
49.88
34,762
59.78
83,363
143.36
445,066
765.37
719,986

Zona "C"
12,250
107.29
15.21
16,882
20.96
19,198
23.84
27,645
34.33
41,314
51.30
45,121

17.27

1,255.23

56.03

Total
35,000
54.45
51,804
81.16
60,206
94.51
118,356
190.50
495,680
832.89
775,015

AO DE
BASE DE FORM.
INICIO DE OPERACIN

1,328.53

daptada al dato base de cada zona


Donde :

Zona B:

G = Consumo por habitante Lit/hab./da


P = Poblacin en miles
K = 204.4

103.89

YECTADOS CON DEMANDA VARIABLE


Zona "A"
5,250
9.15
5,914
10.47
6,246
11.13
7,348
13.36
9,300

Zona C:

(Considerando que la demanda es


Total
AO DE
35,000
BASE DE FORM.
54.45
51,804
INICIO DE OPERACIN
85.42
60,206
101.48
118,356
225.61
495,680

Zona "B"
17,500
30.09
29,008
53.14
34,762
65.13
83,363
174.24
445,066

Zona "C"
12,250
15.21
16,882
21.82
19,198
25.22
27,645
38.01
41,314

17.41

1,146.93

59.72

1,224.07

9,908

719,986

45,121

775,015

18.70

1,970.38

65.95

2,055.03

Zona "B"
13,702
114.29
18.13
18,262
134.86

Zona "C"
7,767
85.83
7.72
13,149
98.71

YECTADOS CON DEMANDA VARIABLE


Zona "A"
4,490
75.32
3.91
5,402
120.50

78.44

(Considerando que la demanda es


Total
x f%Prdidas
25,959
29.76
36,813

#DIV/0!

7.53
5,250
150.63
9.15

28.51
17,500
148.58
30.09

15.02
12,250
107.29
15.21

51.06
35,000

51,804

5,914

29,008

16,882

180.76
12.37
6,246
241.01
17.42
7,348
301.26
25.62
7,348

157.49
52.88
34,762
175.32
70.54
83,363
193.15
186.36
83,363

112.65
22.01
19,198
123.38
27.42
27,645
134.11
42.91
27,645

361.51
30.75
9,300
512.14
55.13
9,908
602.52
69.09
0

204.74
197.55
445,066
233.72
1,203.93
719,986
251.10
2,092.46
0

140.82
45.06
41,314
157.58
75.35
45,121
167.64
87.55
0

723.02
0.00
0
1,024.28
0.00
0
1,205.04
0.00

266.17
0.00
0
303.83
0.00
0
326.43
0.00

176.02
0.00
0
196.98
0.00
0
209.55
0.00

54.45

#DIV/0!

87.26
60,206

#DIV/0!

115.38
118,356

#DIV/0!

254.89
118,356

#DIV/0!

273.36
495,680

#DIV/0!

1,334.41
775,015

#DIV/0!

2,249.10
0

#DIV/0!

0.00
0
0.00
0
0.00

presa + Prdidas + Variac. de consumos

DE INGENIERIA CIVIL Y ARQUITECTURA

ELA ACDEMICA PROFESIONAL DE INGENIERA CIVIL

AMEN PARCIAL(CICLO 1.2010(26/08/10) )


RIO DEL EXAMEN PARCIAL(CICLO 1.2010(26/08/10) )
AGUA Y ALCANTARILLADO - CIC404

HOJA N 07/09

plementarias usaremos los resultados de las estimaciones de los aos hasta el cul el actual sistema cubre las
emanda y las ecuaciones del crecimiento poblacional planteados.

A QUE SON OBRAS DE AMPLIACIN PARA LAS ZONAS A, B Y C, POR LO QUE Xo TIENE UN VALOR VARIABLE PARA CADA CADA CASO UTILIZANDO EL

0.3 (1 - a) Xo ^ 0.85 ) / (r)^ 1/2 ]

.....Frmula General

N.C.D. = NUEVA CAJA DE DISTRIBUCIN

NUEVO R.Z."B" = NUEVA RESERVORIO ZONAL "B"


NUEVO R.Z."C" = NUEVO RESERVORIO ZONAL "C"
AMP.R "A"= AMPLIACION DE REDES DE DISTRIB. EN A
AMP.R "B"= AMPLIACION DE REDES DE DISTRIB. EN B
R.N."C" = REDES NUEVAS PARA "C"

F. ESCAL.

TASA INT.

%COSTO

X1 (S D)

X1(Pond.)

0.140

0.200

0.031

10.979

0.35

0.114
0.282
0.140
0.188
0.140
0.140
0.140
0.250
0.219
0.250

0.200
0.200
0.200
0.200
0.200
0.200
0.200
0.200
0.200
0.200

0.043
0.035
0.031
0.005
0.001
0.772
0.010
0.001
0.067
0.004
1.000

11.352
8.970
10.979
10.295
10.979
10.979
10.979
9.419
9.856
9.419
114.210

0.49
0.31
0.35
0.05
0.01
8.48
0.10
0.01
0.66
0.04
10.84

5.0 %

DE REAJUSTE ANUAL Y EL CRECIMIENTO POBLACIONAL

DE INGENIERIA CIVIL Y ARQUITECTURA

ELA ACDEMICA PROFESIONAL DE INGENIERA CIVIL

AMEN PARCIAL(CICLO 1.2010 (26/08/10) )


RIO DEL EXAMEN PARCIAL(CICLO 1.2010 (26/08/10) )
AGUA Y ALCANTARILLADO - CIC404

HOJA N 08/09

29-2012), QUE ES EL DETERMINADO POR LA DISPONIBILIDAD DE CAUDAL DE LA REPRESA PARA ABASTECER A LAS ZONAS
MODELO LINEAL DE LOS DATOS SUMINISTRADOS INCLUDO PRDIDAS, CORRIGIENDO LOS COSTOS DE OPERACIN CON

POB.(2009)
35,000

COSTO(Dato)
5,000,000

POB.(2026)
495,680

COST.EJ.(Est.)
76,184,955

35,000

700,000

495,680

10,665,894

35,000

6,800,000

495,680

103,611,539

35,000

900,000

495,680

11,846,057

5,500,000
1,550,000

495,680
495,680

83,803,451
23,617,336

N PARALELO(N.L.C.)

MIENTO PARALELO(N.P.T.)
35,000
35,000

COST.MAN(Est.)

L PARALELO(N.R.P.)
35,000
35,000

5,000,000
700,000

495,680
495,680

76,184,955
10,665,894

35,000
35,000

800,000
150,000

495,680
495,680

12,189,593
2,285,549

5,250
5,250

1,328,571
186,000

9,300
9,300

2,724,435
381,421

17,500
17,500

63,580,857
8,901,320

445,066
445,066

1,871,891,384
262,064,794

12,250
12,250

5,902,000
826,280

41,314
41,314

23,042,429
3,225,940

5,250
5,250

3,400,000
850,000

9,300
9,300

3,036,285
759,071

17,500
17,500

5,700,000
1,250,000

445,066
445,066

161,215,898
35,354,364
Ref.:Red A

12,250
12,250

2,800,000
580,000

41,314
41,314

10,931,684
2,264,420

9,338,263
2,334,566

ORICO ANUAL DE LA POBLACIN, TENIENDO EN CUENTA QUE EL SISTEMA ACTUAL SATISFACE A LA POBLACION A, B Y C AL 100% DE SU NECES
t ( a2012 - a2009) / Crecimiento cte.de la demanda anual de diseo =

yecto(Estudio y Ejecucin = 2 aos: 2010 y 2011)

1.20%
11.42%

= r ABC
= r ABC

% incid. =
15.43%
R (crec.anual) =
R (crec.anual) = 790 / 7 aos =

= r ABC

% incid. =
% incid. =

hab.
73.1

INTERCEPTO DE LA
DEMANDA CON EL
EJE DE ABCISAS
(Demanda =0)

Xo A =

% incid. =

57.43%

= r ABC

% incid. =

56.00%

= r ABC

% incid. =
52.18%
R (crec.anual) =
R (crec.anual) = 8454 / 7 aos =

= r ABC

Xo B =

% incid. =

5.33%

= r ABC

% incid. =

32.59%

= r ABC

% incid. =
0.00%
R (crec.anual) =
R (crec.anual) = 22,207 / 22 aos =

= r ABC

Xo C =

Xo prom.=

= r AB

hab.
1,535.1

INTERCEPTO DE LA
DEMANDA CON EL
EJE DE ABCISAS
(Demanda =0)

INTERCEPTO DE LA
DEMANDA CON EL
EJE DE ABCISAS
(Demanda =0)

= r AB

= r AB

hab.
767.4

15

7.00

unid. de tiempo

X''(Pond.)

0.09
0.13
EL SISTEMA PARA LA ZONA SATISFACE POR ENCIMA DEL PERIODOD DE DISEO,
POR LO QUE SERVIRA PARA ABASTECER INCLUSIVE A LA ZONA B Y C A FIN DE
AHORRAR COSTOS EN INFRAESTRUCTURA EXCLUSIVA PARA ESTAS DOS ULTIMAS
ZONAS QUE EN EL 2007 O TIENEN SISTEMA.

0.09
0.09
0.01
0.00
1.45
0.05
0.00
0.11
0.02
2.06

EL SISTEMA PARA LA ZONA SATISFACE POR ENCIMA DEL PERIODOD DE DISEO,


POR LO QUE SERVIRA PARA ABASTECER INCLUSIVE A LA ZONA B Y C A FIN DE
AHORRAR COSTOS EN INFRAESTRUCTURA EXCLUSIVA PARA ESTAS DOS ULTIMAS
ZONAS QUE EN EL 2007 O TIENEN SISTEMA.

USANDO COSTOS DE OPERACIN Y MANTENIMIENTO DE 20 AOS

X1=
X2=
X=

10.84
2.06
12.90

PARA CUBRIR DEMANDA FUTURA (2012-2029)


PARA CUBRIR EL DEFICIT (2012-2009)
REDONDEO =

15

SE ASUME = 15 AOS

CALCULADO CON 15 AOS DE COSTOS DE OPERACIN Y


MANTENIMIENTO.

2012 + 15 AOS = 2026

DE INGENIERIA CIVIL Y ARQUITECTURA

ELA ACDEMICA PROFESIONAL DE INGENIERA CIVIL

RIO DEL EXAMEN PARCIAL(CICLO 1.2010 (26/08/10) )

Montenegro - REG. CIP. 45773


AGUA Y ALCANTARILLADO - CIC404

HOJA N 07/07

s reservorios zonales "A", "B" y "C" se calculan en funcin al cuadro 2 considerando el Consumo promedio para 2005151 l/s. con los datos del cuadro 02 de consumo promedio al tener como dato de persistencia de consumos horarios
terminaron los coeficientes de mayoracin.

ON LA CONSTRUCCIN DE LAS O.C LA PRODUCCIN AL 100%

mos los valores absolutos mximos multiplicado por Cprom.( en m3)

Vi (2000) =

0.0

M3

ES. EFIC. AL 2008 =

83,824

m3 - VOLUMEN DE CONSUMO NETO SIN MAYORACION (2025)

108,972

m3 - VOLUMEN DE CONSUMO NETO CON MAYORACION K1 = 1.3

R. PRINC.COMP.2026

m3-VOLUMEN CONSIDERADO COMO DISEO DE CONSUMO EN 1995

108,972

m3 - VOLUMEN DE CONSUMO NETO CON MAYORACION K1 = 1.3

1,190.81

m3 - RESERV. Zonal "A"

26,004.89

m3 - RESERV. Zonal "B"

1,627.56

m3 - RESERV. Zonal "C"

28,823.26

M3

nal "B" que ser abastecido por el Reservorio Principal


consumo horario se optara por la recomendacin del RNC.

L/s

L/s

L/s

Rpta. H

5 Ptos.

e distribucin se tomaran del cuadro No. 09 para el ao 2021 y el calculo de caudal mximo horario para estimar el k2
nto de tuberas con dimetros y velocidades econmicas, recomendadas por Simn Arocha.(pag. 48)

STIR DATOS DE VARIACIONES DIARIAS DEL AO 2007

INGENICA PARA USAR EL VALOR DE Q MAX. PROM / QPROM QUE EVITARIA SOBREDIMENSIONAR LA TUBERA
3 SE PRESENTAN AL AO CON MENOS DE 30% DE PROBABILIDAD DE OCURRENCIA
ARA EL AO 2040 SE TIENE:
RELACIN DIAMETRO - VELOCIDAD ECONMICA

(Ref. : NORMAS INOS-VENEZOLANAS)

V mx. (m/seg.)

Q mx. (LIT!SEG.

DIMETRO

D = 18"
D = 24"

D=
D=

D=
D=

mm.

Pulg.

75
100
150
200
250
300
350
400
450
500
600
700

3"
4"
6"
8"
10"
12"
14"
16"
18"
20"
24"
30"

0.70
0.75
0.80
0.90
1.00
1.10
1.20
1.25
1.30
1.40
1.60
1.60
Rpta. H

3.05
5.89
14.14
28.27
49.09
77.75
115.45
157.10
206.76
274.90
452.39
729.60

1 Pto.
1 Pto.

Se tiene como datos el consumo promedio


catastral por prdidas es de 80%,75% y 39%.Se
repite con fines acadmicos.

1 Pto.

Se llenar teniendo en cuenta cuadros de ms abajo

1 Pto.

SE USARA LA VARIACION LINEAL POR QUE LA


DIFERENCIA ES SIGNIFICATIVA CON LA
PROYECCION USANDO EL MODELO
MATEMATICO DE CAPEN

Q diseo = 1.3 X

54.45

70.79

VOLUM.
ACUMUL. X 1.3
CONSUMO (2009)
(Lts.)

PRODUCC.PROY.x1.3
ACUM.24Hr.2009
(C /242.19 Lit/seg.)
(Lts.)

VOLUM. DE
CONS.RESERVORIO
Mayorado c/ K1=1,3
(Lts.)

140,157

5,157,360

5,017,203

318,535

10,314,720

9,996,185

675,291

15,472,080

14,796,789

1,337,839

20,629,440

19,291,601

2,025,869

25,786,800

23,760,931

2,713,899

30,944,160

28,230,261

3,414,673

36,101,520

32,686,847

3,937,069

41,258,880

37,321,811

4,523,169

46,416,240

41,893,071

5,262,164

51,573,600

46,311,436

5,848,264

56,730,960

50,882,696

6,115,834

61,888,320

55,772,486

6,115,834

61,888,320

55,772,486
50,755,283

Lit./seg.

Caudal de diseo de la tubera de entrada del Reservorio


Usando la tabla que seala velocidades mximas y gastos mximos admisibles
para cada dimetro, propuesta por las normas Venezolanas (INOS-Ref.3)

39,043

X1.3

50,755.3

12"
REQUERIMIENTO DE DISEO (2009)
54.45 x 1.3 =

70.7851174
12"

ANTERIOR DISEO (2000)

9.15
6"

50,755.3
M3

842.9 X 1.3 =

1,096

Diferencia generada por la produccin, que se entiende fsicamente que si no se cambia el dimetro de la tubera de acceso al Reservorio se tendra que construir un
RESERVORIO de (4,917.2-1,442.0=3,475.20 m3), considerando que en el ao 2000 se uso los mismos % de variaciones de consumo del 2009; si se considera el cambio de tubera
de acceso para transportar 186.30 Lt./s. solo se necesitara u Reservorio de 1, 442 m3 que deber contrastarse con el volumen de consumo del reservorio construdo (
Evaluacin de costos y ventajas tcnicas)

-38,199.6 M3
DEFICIT DE SERVICIO EN EL AO 2009 AL PRODUCIR 151 Lit./ seg. PARA CUBRIR UNA DEMANDA DE
186.30 Lit./s. NETA DE CONSUMO POR EDIFICACIN SIN CONSIDERAR
PERDIDAS+CONS.PBLICO+OPERAC.YMANTEN.VARIAC. DE DOTACIN CON EL TIEMPO.

VARIA NOTORIAMENTE AL INCREMENTARSE LOS % DE VARIACIONDE CONSUMO MANTENIENDOSE EL Q=186.3 Lit/seg. Cte. com
842.9

2 Ptos.

<

38,670.00

EL VOLUMEN DEL RESERVORIO EN EL 2031 SERA:


842.9

1.3

1,095.8 M3

2 Ptos.

Qdisponible sistema includo represa

Corregido por Capen

Qdisponible sistema includo represa


551+1000 = 1,001 l/s.

Qdisponible sistema includo represa


151 + 850 = 1,001 l/s.
0.00 POR AO
0.00 PARA 2066
0.00 PARA 2063

0.00 POR MES


0.00 JULIO
0.00 AGOSTO

#DIV/0!

En los proyectos q estamos eval no es lineal


Ya no retrocedemos hasta 2 habitantes.

Para fines academiscos es lineal.

6 AOS MENOS QUE EL CUADRO DE USO CONSTANTE DE LA DEMAN

#DIV/0!

POR AO

#DIV/0!
#DIV/0!

PARA 2028
PARA 2029

#DIV/0!
#DIV/0!
#DIV/0!

POR MES
Feb-29
Mar-29

Qdisponible sistema includo represa, considerando variacin tipificada socio econmica de la demanda en el tiempo + p+erdidas en la red
151 + 850 = 1,001 l/s.
Qdisponible sistema includo represa, considerando slo variacin tipificada socio econmica de la demanda en el tiempo
151 + 850 = 1,001 l/s.
-1,124.55 POR AO
1,124.55 PARA 2030
0.00 PARA 2031

-93.71 POR MES


41 AOS MENOS QUE EL CUADRO DE USO CONSTANTE DE LA DEMAN
1,030.84
Jan-31 35 AOS MENOS QUE EL CUADRO DE USO VARIABLE DE LA DEMAND
937.13

Feb-31

AO FINAL DE
EFICIENCIA DEL

39 AOS MENOS QUE EL CUADRO DE USO CONSTANTE DE LA DEMAN

2 Ptos.

32 AOS MENOS QUE EL CUADRO DE USO VARIABLE DE LA DEMAND

DATOS INICIALES DE COSTOS DE CONSTRUCCION Y OPERACIN (2008)

No. ORDEN

ESTRUCT.
COMPLEM.

01

CAPTACIN

02

LNEA DE CONDUCCIN

03

PLANTA DE TRATAMIENTO

04

RESERVORIO PRINCIPAL

05

CAJA DE DISTRIBUCIN

06

REDES PARA "A"

07

REDES PARA "B"

08

REDES PARA "C"

09

C. EJEC.

C.OP.yMANT.

( S/.)

( S/.)

5,000,000
6,800,000
5,500,000
5,000,000
800,000
3,400,000
5,700,000
2,800,000

700,000
900,000
1,550,000
700,000
150,000
850,000
1,250,000
580,000

35,000,000

6,680,000
19.09%

10
11

AO PARA EL QUE FUE


EFICIENTE EL SISTEMA
CUBRE LA DEMANDA EL
SAP QUE ENTRO EN
OPERACIN EL AO 199O Y
CUYO Pd=20 aos, hasta el
2010.

#REF!

FIN DEL PERIODO DE DISEO SUPUESTO DE FORMA PRELIMINAR

COSTO PROYECTADO AL
2012 (Ao en que entra en

Interes simple:
Interes compuesto:

C = Co ( 1 + r (t-to) )

T-TO =

C = Co . r (t-to)
2012

2009 =
COSTOS DE CONSTRUCCION

3 aos

COSTOS
PROPORCIONA
LES A LAS
OBRAS
EXISTENTES
DISEADAS EL
2000, EN
FUNCION A LA
POBLACION
SERVIDA
CUYOS
VALORES SE
DISTORSIONAN
EN BASE A LOS
OBTENIDOS
DEL
PLANEAMIENTO
INICIAL

COSTOS
REALES DE
EJECUCIN Y
MANTENIMIENT
OS DE LAS
REDES EN A Y B
SEGN
PLANEAMIENTO
DE DISEO
Ref.:Red B

15,655,324
3,433,185

COMO LA
REPRESA
ABASTECE AL
2029 NO SERA
NECESARIO
UBICAR OTRA
FUENTE PARA
DIMENSIONAR
HASTA EL 2027.
DEBERA
PROGRAMARSE
LA EJECUCIN
POR ETAPAS EL
PROYECTO DE
AMPLIACION EN
FUNCION A UN
RECALCULO
POBLACIONAL
EN BASE A UN
ESTUDIO DE
RENTABILIDAD(
Recomendacin
: Encontrar el
punto de
inflexin de
densidad y
determinar la
velocidad de
crecimiento para
determinar la
poblacin
mxima

O SIN MAYORACION (2025)

O CON MAYORACION K1 = 1.3

O DISEO DE CONSUMO EN 1995 CONSIDERANDO FACTOR DE MAYORACION K1=1.3(2,523X1.3)

O CON MAYORACION K1 = 1.3

EVALUAR SU APLICACIN COMPARANDO CON EL METODO DIRECTO Y EL ANALIS

632.13 EVALUAR SU APLICACIN COMPARANDO CON EL METODO DIRECTO Y EL ANALIS


USAR ESTE VALOR O.K.

ESTOS
VALORES LOS
COMPARAMO

CONSUMO DETERMINADO EN EL EXPEDIENTE DEL PROYECTO PARA DETERMINAR EL VOLUMEN DEL RESERVORIO EXISTENTE ( 2000) CON PD=

GASTO

ntrada del Reservorio

imas y gastos mximos admisibles


as Venezolanas (INOS-Ref.3)

l/s

l/s

M3

o se tendra que construir un


09; si se considera el cambio de tubera
o del reservorio construdo (

HORAS

CUADRO 03.-

PROM.(1999)
@ 2 horas
% del Qprom.

DE LA LOCAL.

(Lt./seg.)

(Lts./s.)

LOCALIDAD (Consumo prom.usado en el diseo del proyecto(2000) con variacione


del ao 1999 para las zonas A y B Pd=15 aos (2015))

GASTO
PROM @ 2HR.

VOLUM.
VOLUMEN

(Lts.)

PRODUCC. PROY.

VOLUM. DE

ACUMUL.

ACUM.24Hr.2015

CONS.RESERVORIO

CONSUMO (2015)

(C / 151Lit/seg.)

100% proy.demanda.2015

(Lts.)

(Lts.)

(Lts.)

0-2

5.00%

0.000

2-4

15.00%

0.000

4-6

25.00%

0.000

6-8

85.00%

0.000

8-10

155.00%

0.000

10-12

125.00%

0.000

12-14

270.00%

0.000

14-16

150.00%

0.000

16-18

105.00%

0.000

18-20

145.00%

0.000

20-22

70.00%

0.000

22-24

50.00%

0.000

0-2
POR DA

5.00%

0.000
0.00

0
0
0.00 Reserv(343.68 l/s.)=

(Lts./s.)

0
(Lts.)

(M3)

Vol.diseo inic.reservorio 95:

0.00

Volumen de consumo del Reservorio si se utiliza los % de variacin del 2009


DIFERENCIA

0.0
842.9
842.9

DECIDIMOS INCREMENTAR EN FORMA PARALELA UNA LINEA DE CONDUCCIN PARA INCREMENTAR EL CAUDAL DE 151 A 186.3, Qd= 35.3 Lt.
POR LO QUE NECESITAMOS INCREMENTAR UN RESERVORIO DE 328 M3 PARA CUBRIR LA NECESIDAD SOLO GENERADA AL 2009
NECESITAMOS CALCULAR LA NECESIDAD PARA EL AO 2031 CONSIDERANDO UN PERIODO DE DISEO INICIAL DE 20 AOS

IENDOSE EL Q=186.3 Lit/seg. Cte. como produccin


Q demanda mxima horaria = 201.83

DE USO CONSTANTE DE LA DEMANDA

anda en el tiempo + p+erdidas en la red

DE USO CONSTANTE DE LA DEMANDA


DE USO VARIABLE DE LA DEMANDA USANDO EL MODELO DE CAPEN

DE USO CONSTANTE DE LA DEMANDA


DE USO VARIABLE DE LA DEMANDA USANDO EL MODELO DE CAPEN

EL METODO DIRECTO Y EL ANALISIS ECONOMICO DE LA OBRA

EL METODO DIRECTO Y EL ANALISIS ECONOMICO DE LA OBRA

L RESERVORIO EXISTENTE ( 2000) CON PD=15 Aos

CONSUMO DETERMINADO EN BASE A LA PROYECCIN DE VARIACIONES DE CONSUMO Y EL CAUDAL DE DISEO P


Q dis( 2026) =

o del proyecto(2000) con variaciones de consumo


as A y B Pd=15 aos (2015))

(C / 186.30 Lit/seg.)

GASTO

CUADRO 03.-

100% demanda.2008

PROM.(2031)
@ 2 horas
% del Qprom.

DE LA LOCAL.

(Lts.)

(Lt./seg.)

(Lts./s.)

CONSUMO REAL
ACUM.24Hr.2009

55.13 litros/seg.

VOLUM. DE

HORAS

CONS.RESERVORIO

(Lts.)

LOCALIDAD (Consumo prom.usado en el diseo del proyecto(2000) con va


Pd=15 aos (2015))

GASTO
PROM @ 2HR.

VOLUM.
VOLUMEN

ACUMUL.
CONSUMO (2008)

(Lts.)

(Lts.)
0

107,813

-107,813

0-2

60.00%

33.078

248,087

248,087

245,027

-245,027

2-4

65.00%

35.835

267,934

516,020

519,455

-519,455

4-6

70.00%

38.591

307,627

823,648

1,029,107

-1,029,107

6-8

85.00%

46.861

406,861

1,230,509

1,558,361

-1,558,361

8-10

120.00%

66.156

466,402

1,696,910

2,087,615

-2,087,615

10-12

115.00%

63.400

516,020

2,212,931

2,626,672

-2,626,672

12-14

145.00%

79.939

535,867

2,748,798

3,028,515

-3,028,515

14-16

125.00%

68.913

466,402

3,215,200

3,479,361

-3,479,361

16-18

110.00%

60.643

446,555

3,661,754

4,047,819

-4,047,819

18-20

115.00%

63.400

426,708

4,088,462

4,498,665

-4,498,665

20-22

100.00%

55.130

377,089

4,465,552

4,704,488

-4,704,488

22-24

90.00%

49.617

297,702

4,763,254

4,704,488
Reserv(343.68 l/s.)=

-4,704,488
3,979,210

0-2

60.00%

33.078
661.56

4,763,254

4,763,254
4,763.25

POR DA

Vol.diseo ampliac.reserv.2008

M3

3,979.2
M3
M3

L CAUDAL DE 151 A 186.3, Qd= 35.3 Lt./seg.


SOLO GENERADA AL 2009
INICIAL DE 20 AOS

(Lts./s.)

(Lts.)

(M3)

55.13

Se calcula con ste dato (2031) por encontarse cercano al ao del periodo de diseo calculado de 15

VARIACIONES DE CONSUMO Y EL CAUDAL DE DISEO PARA EL AO 2031, USANDO PD=20 Aos (2012-2031)

RELACIN DIAMETRO - VELOCIDAD ECONMICA

LIDAD (Consumo prom.usado en el diseo del proyecto(2000) con variaciones de consumo del ao 1999 para las zonas A y B
Pd=15 aos (2015))
PRODUCC. PROY.

VOLUM. DE

CONSUMO REAL

ACUM.24Hr.2008

CONS.RESERVORIO

ACUM.24Hr.2009

(C / 151Lit/seg.)

100% proy.demanda.2015

(C / 186.30 Lit/seg.)

(Lts.)

(Lts.)

(Lts.)

(Ref. : NORMAS INOS-VENEZOLANAS)


DIMETRO

VOLUM. DE
CONS.RESERVORIO
100% demanda.2008

(Lts.)

mm.

Pulg.

75
100

3"
4"

150

6"

396,938

148,851

396,938

200

8"

793,876

277,855

793,876

250

10"

1,190,813

367,166

1,190,813

300

12"

1,587,751

357,242

1,587,751

350

14"

1,984,689

287,779

1,984,689

400

16"

2,381,627

168,696

2,381,627

450

18"

2,778,565

29,767

2,778,565

500

20"

3,175,502

-39,697

3,175,502

600

24"

3,572,440

-89,314

3,572,440

750

30"

3,969,378

-119,084

3,969,378

4,366,316

-99,236

4,366,316

4,763,254

4,763,254

4,763,254
4,763,254

4,763,254
Reserv(1003.28 l/s.)=

486,250

Vol.diseo inic.reservorio 95:

Reserv(343.68 l/s.)=

Vol.diseo ampliac.reserv.2008

486.25 M3

x 1.3 =

cano al ao del periodo de diseo calculado de 15 aos (2012 + 15 =2027)

632.13 M3

ELACIN DIAMETRO - VELOCIDAD ECONMICA


(Ref. : NORMAS INOS-VENEZOLANAS)
V mx. (m/seg.)

Q (m3/seg.)

0.70
0.75

3.05
5.89

0.80

14.14

0.90

28.27

1.00

49.09

1.10

77.75

1.20

115.45

1.25

157.10

1.30

206.76

1.40

274.90

1.60

452.39

1.60

729.60

FACULTAD DE INGENIERIA CIVIL Y ARQUITECTURA


ESCUELA ACDEMICA PROFESIONAL DE INGENIERA CIVIL

SOLUCIONARIO DEL EXAMEN DE SUBSANACIN(CICLO 2.2009 (14/03/10) )


RESP.DEL REA HIDRAULICA :
ASIGNATURA

Ing. Civil Clifton Paucar y Montenegro - REG. CIP. 45773


ABASTECIMIENTOS DE AGUA Y ALCANTARILLADO - CIC404

HOJA N 01/09

RPTA A.-

i)

Iniciamos el proceso de estimacin calculando la dotacin promedio CATASTRAL, en funcin al consumo


promedio cuantificado por tipo de EDIFICACIN Y HABITANTE, as como el consumo promedio en Lt./seg.
80,500

Poblacin total localidad ALFA

Habitantes

ZONA A :

Densidad poblacional promedio :

DATOS

TIPO1( unid.)=

1,000

TIPO2( unid.) =

885

TIPO3( unid.) =

500

TIPO4( unid.) =

300

Lit./viv./da

Lit./hab./da

Lit./viv./da

Lit./hab./da

Lit./viv./da

Lit./hab./da

Lit./viv./da

Lit./hab./da

430
345
380

2,230
1,790
2,020

370
295
325

1,400
1,200
1,302

230
188
206

1,250
940
1,120

206
155
195

Mximo
2,600
Mnimo
2,100
Promedio
2,300
TOTAL VIVIENDAS

2,685

Consumo promedio por edificacin/ da =

Poblacin en el Ao 2009:

(RED EXISTENTE)

6.00 habitantes por vivienda

1,000 x 2,300 885 x 2,020 500 x 1,250 300 x 1,118


(1,000865 500300 )

20.00%

80,500 Hab.

Densidad poblacional promedio por edificacin


Dotacin para el AO 2009 en base al consumo por tipo de edificacin

1,890.02 Lit./edif./da

16,100 hab.

6.00 hab

315.00 Lt/hab./da

(x C.R:Includo 17% de prdidas y desperdicios en la red de reciente instalacin)

Dotacin para el AO 2009 en base al consumo por habitante

308.80 Lt/hab./da

(xC.C:Incidencia futura de incremento de consumo por la tendencia de crecimiento comercial)

TOMANDO EL VALOR MAYOR DE DOTACIN TENDREMOS


Consumo promedio instantaneo para la zona A

58.70 Lt./seg.

(RED EXISTENTE)

ZONA B :

Densidad poblacional promedio :

DATOS

TIPO1( unid.)=

2000

TIPO2( unid.) =

1500

TIPO3( unid.) =

800

TIPO4( unid.) =

620

Lit./viv./da

Lit./hab./da

Lit./viv./da

Lit./hab./da

Lit./viv./da

Lit./hab./da

Lit./viv./da

Lit./hab./da

240
190
200

1,810
1,450
1,648

200
160
180

1,750
1,220
1,370

190
135
150

1,400
1,040
1,141

150
115
128

Mximo
2,200
Mnimo
1,750
Promedio
1,850
TOTAL VIVIENDAS

4,920

Consumo promedio por edificacin/ da =


Poblacin en el Ao 2009:

9.00 habitantes por vivienda

2,000 x1,850 1,500 x 1,648 800x 1,370 620x 1,141


( 2,0001,500 800 620 )

55.00%

80,500 Hab.

Densidad poblacional promedio por edificacin


Dotacin para el AO 2009 en base al consumo por tipo de edificacin

1,621.02 Lit./edif./da

44,275 hab.

9.00 hab

180.11 Lt/hab./da

(x C.R:Includo 45% de prdidas y desperdicios en la red de instalacin antigua)

Dotacin para el AO 2009 en base al consumo por habitante

176.70 Lt/hab./da

(xC.C:Incidencia futura de incremento de consumo al mantener la tendencia de crecimiento sostenido de tipo residencial)

TOMANDO EL VALOR MAYOR DE DOTACIN TENDREMOS


Consumo promedio instantaneo para la zona B

92.30 Lt./seg.

(RED EN PROYECCIN)

ZONA C :

Densidad poblacional promedio :

DATOS

TIPO1( unid.)=

1300

TIPO2( unid.) =

1800

TIPO3( unid.) =

1200

TIPO4( unid.) =

730

Lit./viv./da

Lit./hab./da

Lit./viv./da

Lit./hab./da

Lit./viv./da

Lit./hab./da

Lit./viv./da

Lit./hab./da

700
600
650

200
165
180

700
550
600

190
150
165

700
450
500

180
125
132

550
350
350

140
90
100

Mximo
Mnimo
Promedio

4.00 habitantes por vivienda

FACULTAD DE INGENIERIA CIVIL Y ARQUITECTURA


ESCUELA ACDEMICA PROFESIONAL DE INGENIERA CIVIL

SOLUCIONARIO DEL EXAMEN DE SUBSANACIN(CICLO 2.2009 (14/03/10) )


RESP.DEL REA HIDRAULICA :

Ing. Civil Clifton Paucar y Montenegro - REG. CIP. 45773

ABASTECIMIENTOS DE AGUA Y ALCANTARILLADO - CIC404

ASIGNATURA

Consumo promedio por edificacin/ da =

Poblacin en el Ao 2009:

1,300 x 650 1,800 x 600 1,200 x 500 730 x 350


(1,300 1,800 1,200 730 )

25.00%

80,500 Hab.

Densidad poblacional promedio por edificacin


Dotacin para el AO 2009 en base al consumo por tipo de edificacin

HOJA N 02/09

552.78 Lit./edif./da

20,125 hab.

4.00 hab

138.20 Lt/hab./da

(x C.R:Includo 17% de prdidas y desperdicios en la red de reciente instalacin)

Dotacin para el AO 2009 en base al consumo por habitante

151.57 Lt/hab./da

(xC.C:Incidencia futura de incremento de consumo por la tendencia de crecimiento comercial)

TOMANDO EL VALOR MAYOR DE DOTACIN TENDREMOS


Consumo promedio instantaneo para la zona C

35.30 Lt./seg.

Rpta. A y B
ZONA CON SERVICIO DE AGUA

ZONA SIN SERVICIO DE AGUA

CUADRO DE RESPUESTAS A Y B.DEMANDA CATASTRAL PROMEDIO:

Zona "A"

Zona "B"

Zona "C"

Densidad prom.poblac.(hab./edif.)

6.00

9.00

4.00

Poblacin Total ao 2009 ( habitantes )


Dotacin para el ao 2009 (Lt./hab./da)

16,100
315.00

44,275
180.11

20,125
151.57

80,500.00

Consumo prom.ao 2009 ( Lt./seg. )

58.70

92.30

35.30

186.30

Q de diseo del sistema ao 2000(Dato)

Total

DEFICIT./SUPERAV.

123.4%

151.00

Rpta. C
CUADRO DE RESPUESTA PARA EL INCISO C.DEMANDA CATASTRAL POR EDIFICACIN
Includo/PRDIDAS:

Zona "A"

Zona "B"

Zona "C"

6.00

9.00

4.00

Poblacin Total ao 2009 ( habitantes )

16,100

44,275

20,125

Dotacin para el ao 2009 calculada por


edificacin sin prdidas(Lt./hab./da)

315.00

180.11

138.20

Dotacin para el ao 2009 Inc. Prdidas segn datos


suministrados 60%-45%-17% (Lt./hab./da)
Dotacin para el ao 2015 Inc. Prdidas segn datos
proyectados 60%-45%-17% (Lt./hab./da)
Dotacin para el ao 2021 Inc. Prdidas segn datos
proyectados 60%-45%-17% (Lt./hab./da)
Dotacin para el ao 2031 Inc. Prdidas segn datos
proyectados 60%-45%-17% (Lt./hab./da)

504.00

261.16

161.69

504.00

261.16

161.69

504.00

261.16

161.69

504.00

261.16

161.69

Consumo prom.ao 2009 sin / Prdidas


(Lt./seg. )
Consumo prom.ao 2009 includo/ Prdidas
(Lt./seg. )

58.70

92.30

32.19

183.19

93.92

133.83

37.66

265.41

Poblacin proyectada ao 2015

0.00

0.00

0.00

0.00

Consumo prom.ao 2015 sin / Prdidas


(Lt./seg. )
Consumo prom.ao 2015 Includo / Prdidas
(Lt./seg. )

0.00

0.00

0.00

0.00

0.00

0.00

0.00

0.00

Poblacin proyectada ao 2021

0.00

0.00

0.00

0.00

Consumo prom.ao 2021 sin / Prdidas


(Lt./seg. )
Consumo prom.ao 2021 Includo / Prdidas
(Lt./seg. )

0.00

0.00

0.00

0.00

0.00

0.00

0.00

0.00

Poblacin proyectada ao 2031

0.00

0.00

0.00

0.00

Consumo prom.ao 2031 sin / Prdidas


(Lt./seg. )
Consumo prom.ao 2031 Includo / Prdidas
(Lt./seg. )

0.00

0.00

0.00

0.00

0.00

0.00

0.00

0.00

#DIV/0!

PROM.:

#DIV/0!

Densidad prom.poblac.(hab./edif.)

Q de diseo del sistema ao 2000(Dato)

151.00

Total

80,500.00

144.9%

#DIV/0!

#DIV/0!

EL ANLISIS DE LA VARIACIN DE CONSUMO EN EL TIEMPO SE EFECTUAR CON LA DECISIN DE USAR PARA EL CALCULO DE LA
DOTACIN INCREMENTADA , UNA ECUACIN LINEAL CON LOS DATOS (ZONAS URBANAS-DONDE EL INCREMENTO DEL CONSUMO NO
SOLO DEPENDE DEL CRECIMIENTO POBLACIONAL) O EL MODELO MATEMATICO DE CAPEN (ZONAS RURALES-DONDE LA DOTACIN
SE BASA EN EL USO DOMESTICO Y SU VARIACIN DEPENDE DEL INCREMENTO POBLACIONAL), PARA LO CUL ANTES DEBEMOS
ESTIMAR EL QPROMEDIO DE DISEO SEGN LAS CARACTERISTICAS DE CONSUMO DIARIO Y HORARIO, LOS FACTORES DE
MAYORACIN K1 Y K2 Y LA POBLACIN FUTURA

ii)

Efectuamos las estimaciones de las demanda por variaciones de consumo diarias en funcin a los datos
suministrados del diario promedio horario y del diario maximo horario en Lt./seg., a fin de determinar los
valores de K2.

Para determinar el caudal medio en Lt./seg. durante el ao 2009, se proceder a desarrollar el cuadro que contiene las variaciones horarias del
consumo promedio durante el ao, que representa en forma analitica a la curva masa (Produccin y Demanda vrs. Tiempo), teniendo como base que
se consume 151.00 Lt./seg. (2009), y que es la produccin total de la captacin (100%) que es de 151 Lt.seg. con el cul se diseo el reservorio en el
ao 2000, proyecto que entr en funcionamiento el ao 2001 con un Pd=15 aos, debiendo cumplir con eficiencia la demanda hasta el ao 2015,
PERO SE OBSERVA EN EL CUADRO 01 que slo cumple con la demanda del ao 2009 PARA LAS ZONA A Y B, Y QUE DE INCLUIRSE LA
DEMANDA INSATISFECHA EN C QUE NO CUENTA CON SISTEMA, EXISTE UN DEFICIT EN ESTE AO 2009 DE 44.4% .

DATO:

HORAS

Qdisp. Mx.=

1,001 Litros/seg.

LOCALIDAD ( Consumo promedio ao 2009 )

GASTO

CUADRO 02.-

PROM.
@ 2 horas
% del Qprom.

GASTO

VOLUM.(2009)

PRODUCC.(2009)

VOLUM. DE

SIMULACIN

VOLUM. DE

PROM @ 2HR.

VOLUMEN

ACUM.CONSUMO

ACUM.24Hr.

CONS.RESERV.

PRODUCC.(2009)

CONS.RESERV.

(186.30 Lt./seg.)

(186.30 Lt./seg.)

(186.30 Lt./seg.)

(151 Lt./seg.)

(Demanda 186.3)

(186.30 Lt./seg.)

(Demanda 186.30 Lt./s)

(Lt./seg.)

(Lts./s.)

(Lts.)

(Lts.)

(Lts.)

(Lts.)

(Lts.)

(Lts.)

25.00%

46.575

368,874

368,874

1,087,200

718,326

1,341,360

972,486

30.00%

55.890

469,476

838,350

2,174,400

1,336,050

2,682,720

1,844,370

40.00%

74.520

938,952

1,777,302

3,261,600

1,484,298

4,024,080

2,246,778

100.00%

186.300

1,743,768

3,521,070

4,348,800

827,730

5,365,440

1,844,370

160.00%

298.080

1,810,836

5,331,906

5,436,000

104,094

6,706,800

1,374,894

10

110.00%

204.930

1,810,836

7,142,742

6,523,200

-619,542

8,048,160

905,418

12

160.00%

298.080

1,844,370

8,987,112

7,610,400

-1,376,712

9,389,520

402,408

14

115.00%

214.245

1,374,894

10,362,006

8,697,600

-1,664,406

10,730,880

368,874

16

90.00%

167.670

1,542,564

11,904,570

9,784,800

-2,119,770

12,072,240

167,670

18

140.00%

260.820

1,944,972

13,849,542

10,872,000

-2,977,542

13,413,600

-435,942

20

150.00%

279.450

1,542,564

15,392,106

11,959,200

-3,432,906

14,754,960

-637,146

22

80.00%

149.040

704,214

16,096,320

13,046,400

-3,049,920

16,096,320

24

25.00%

46.575

16,096,320

13,046,400

POR DA

2,235.60
(Lts./s.)

16,096,320

16,096.32

(Lts.)

(M3)

186.30

k2prom.09:

Reserv.151 l/s.

16,096,320
4,917,204

Vol.diseo inic.reservorio 95:

Reserv(186.3 l/s.)=

1,441,962

Vol.diseo ampliac.reservor 2008

1.60

VALOR SIN APLICACIN DEL FACTOR DE MAYORACION K1 =1.3


4,917.2
M3
1,442.0
Como se tiene como dato que la persistencia de consumo de los valores mximos diarios es de 10 a 13% se utilizar el cuadro No. 02 para estimar el
valor de K2 y el Volumen de consumo del reservorio principal de ampliacin. Se muestra los graficos de las curvas masa en ambos casos con fines
de comparacin de los consumos.

FACULTAD DE INGENIERIA CIVIL Y ARQUITECTURA


ESCUELA ACDEMICA PROFESIONAL DE INGENIERA CIVIL

SOLUCIONARIO DEL EXAMEN DE SUBSANACIN(CICLO 2.2009 (14/03/10) )


RESP.DEL REA HIDRAULICA :
ASIGNATURA

50,000,000
45,000,000
40,000,000

35,000,000
30,000,000
25,000,000

Ing. Civil Clifton Paucar y Montenegro - REG. CIP. 45773


ABASTECIMIENTOS DE AGUA Y ALCANTARILLADO - CIC404

HOJA N 03/09

25,000,000
20,000,000

15,000,000
10,000,000
5,000,000
0

10

12

14

16

18

20

22

SIMULACIN PRODUCC.(2009) (54.45 Lt./seg.) (Lts.)


PRODUCC.(2009) ACUM.24Hr. (9.5 Lt./seg.) (Lts.)

SIMULACIN PRODUCC.(2009) (54.45 Lt./seg.) (Lts.)


PRODUCC.(2009) ACUM.24Hr. (54.45Lt./seg.) (Lts.)

VOLUM. ACUMUL. Cons.da mx. (Lts.)

HORAS

GASTO
CUADRO 03.PROM.
GASTO
@ 2 horas
PROM @ 2HR.
% del Qprom.
(186.30 Lt./seg.)
(Lt./seg.)

(Lts./s.)

LOCALIDAD (Consumo del da mximo ao 2009 )


VOLUM.

PRODUCC.(2009)

VOLUM. DE

VOLUMEN

ACUMUL.

ACUM.24Hr.

CONSUM.RES. (100%)

(186.30 Lt./seg.)

Cons.da mx.

(186.3 Lt./seg.)

Cons.da mx.

(Lts.)

(Lts.)

(Lts.)

(Lts.)

30.00%

55.89

435,960

435,960

1,087,200

651,240

35.00%

65.21

536,580

972,540

2,174,400

1,201,860
1,283,004

45.00%

83.84

1,006,056

1,978,596

3,261,600

105.00%

195.62

1,810,872

3,789,468

4,348,800

559,332

165.00%

307.40

1,911,456

5,700,924

5,436,000

-264,924

10

120.00%

223.56

2,146,176

7,847,100

6,523,200

-1,323,900

12

200.00%

372.60

2,112,660

9,959,760

7,610,400

-2,349,360

14

115.00%

214.25

1,408,464

11,368,224

8,697,600

-2,670,624

16

95.00%

176.99

1,609,668

12,977,892

9,784,800

-3,193,092

18

145.00%

270.14

2,045,592

15,023,484

10,872,000

-4,151,484

20

160.00%

298.08

1,643,184

16,666,668

11,959,200

-4,707,468

22

85.00%

158.36

771,300

17,437,968

13,046,400

-4,391,568

24

30.00%

55.89

17,437,968

13,046,400

-4,391,568

17,437.97

Reserv(186.3 l./s.)=

5,990,472

POR DA

2,421.94
(Lts./s.)

Rpta. D:

k1=

1.30

17,437,968
(Lts.)

(M3)

Vol.diseo ampliac.reservor 2008 por cons.mx.horar.

201.83

k2dis.(2008):

2.00 (Coef.da de mx.consumo)

(Reglam.)

k2dis.09:

1.60 (Consumo Promedio)

5,990.00

M3

1 Pto.
1 Pto.

1 Pto.

1 Pto.

SE USARA LA VARIACION LINEAL POR


QUE LA DIFERENCIA ES SIGNIFICATIVA
CON LA PROYECCION USANDO EL
MODELO MATEMATICO DE CAPEN

Q diseo = 1.3 X

186.30

242.19

VOLUM.

PRODUCC.PROY.x1.3

VOLUM. DE

Lit./seg.

ACUMUL. X 1.3
CONSUMO (2009)

ACUM.24Hr.2009
(C /242.19 Lit/seg.)

(Lts.)

(Lts.)

Caudal de diseo de la tubera de entrada del Reservorio

CONS.RESERVORIO

Usando la tabla que seala velocidades mximas y gastos mximos admisibles para
cada dimetro, propuesta por las normas Venezolanas (INOS-Ref.3)

Mayorado c/ K1=1,3

(Lts.)

=
479,536

1,413,360

933,824

1,089,855

2,826,720

1,736,865

2,310,493

4,240,080

1,929,587

4,577,391

5,653,440

1,076,049

6,931,478

7,066,800

135,322

9,285,565

8,480,160

-805,405

11,683,246

9,893,520

-1,789,726

13,470,608

11,306,880

-2,163,728

15,475,941

12,720,240

-2,755,701

18,004,405

14,133,600

-3,870,805

20,009,738

15,546,960

-4,462,778

20,925,216

16,960,320

-3,964,896

20,925,216

16,960,320

4,917 X1.3

20"
REQUERIMIENTO DE DISEO (2009)
186.30 x 1.3 =
242.19 l/s
20"

ANTERIOR DISEO (2000)

151.00 l/s
16"

-3,964,896

6,392,365
6,392.4 M3

6,392.4

1,442.0 X 1.3 =

1,875 M3

Diferencia generada por la produccin, que se entiende fsicamente que si no se cambia el dimetro de la tubera de acceso al Reservorio se tendra que
construir un RESERVORIO de (4,917.2-1,442.0=3,475.20 m3), considerando que en el ao 2000 se uso los mismos % de variaciones de consumo del 2009; si se
considera el cambio de tubera de acceso para transportar 186.30 Lt./s. solo se necesitara u Reservorio de 1, 442 m3 que deber contrastarse con el volumen de
consumo del reservorio construdo ( Evaluacin de costos y ventajas tcnicas)

-3,475.2 M3
DEFICIT DE SERVICIO EN EL AO 2009 AL PRODUCIR 151 Lit./ seg. PARA CUBRIR UNA DEMANDA DE 186.30 Lit./s. NETA DE
CONSUMO POR EDIFICACIN SIN CONSIDERAR PERDIDAS+CONS.PBLICO+OPERAC.YMANTEN.VARIAC. DE DOTACIN
CON EL TIEMPO.

DECIDIMOS INCREMENTAR E
POR LO QUE NECESITAMOS
NECESITAMOS CALCULAR LA

M3

2 Ptos.

VARIA NOTORIAMENTE AL INCREMENTARSE LOS % DE VARIACIONDE CONSUMO MANTENIENDOSE EL Q=186.3 Lit/seg. Cte. como
1,442.0
5,990.00
Q demanda mxima horaria = 20
<
EL VOLUMEN DEL RESERVORIO EN EL 2031 SERA:
1,442.0

1.3

1,874.6 M3

CONSUMO DETERMINADO EN EL EXPEDIENTE DEL PROYECTO PARA DETERMINAR EL VOLUMEN DEL RESERVORIO EXISTENTE ( 2000) CON PD=15 Aos

GASTO
HORAS

CUADRO 03.-

PROM.(1999)
@ 2 horas
% del Qprom.

DE LA LOCAL.

(Lt./seg.)

(Lts./s.)

LOCALIDAD (Consumo prom.usado en el diseo del proyecto(2000) con variaciones de consumo del ao 1999 para las zonas A y B
Pd=15 aos (2015))

GASTO
PROM @ 2HR.

VOLUM.
VOLUMEN

(Lts.)

PRODUCC. PROY.

VOLUM. DE

CONSUMO REAL

ACUMUL.

ACUM.24Hr.2015

CONS.RESERVORIO

ACUM.24Hr.2009

CONSUMO (2015)

(C / 151Lit/seg.)

100% proy.demanda.2015

(C / 186.30 Lit/seg.)

(Lts.)

(Lts.)

(Lts.)

(Lts.)

0-2

5.00%

7.550

108,720

108,720

1,087,200

978,480

2-4

15.00%

22.650

217,440

326,160

2,174,400

1,848,240

368,874
838,350

4-6

25.00%

37.750

597,960

924,120

3,261,600

2,337,480

1,777,302

6-8

85.00%

128.350

1,304,640

2,228,760

4,348,800

2,120,040

3,521,070

8-10

155.00%

234.050

1,522,080

3,750,840

5,436,000

1,685,160

5,331,906

10-12

125.00%

188.750

2,147,220

5,898,060

6,523,200

625,140

7,142,742

12-14

270.00%

407.700

2,283,120

8,181,180

7,610,400

-570,780

8,987,112

14-16

150.00%

226.500

1,386,180

9,567,360

8,697,600

-869,760

10,362,006

16-18

105.00%

158.550

1,359,000

10,926,360

9,784,800

-1,141,560

11,904,570

18-20

145.00%

218.950

1,168,740

12,095,100

10,872,000

-1,223,100

13,849,542

20-22

70.00%

105.700

652,320

12,747,420

11,959,200

-788,220

15,392,106

22-24

50.00%

75.500

298,980

13,046,400

13,046,400

16,096,320

0-2

5.00%

7.550

13,046,400

13,046,400

13,046.40

Reserv(343.68 l/s.)=

1,114,380

Reserv(343.68 l/s.)=

POR DA

1,812.00
(Lts./s.)

13,046,400
(Lts.)

(M3)

151.00

Volumen de consumo del Reservorio si se utiliza los % de variacin del 2009


DIFERENCIA

16,096,320

Vol.diseo inic.reservorio 95:

Vol.diseo ampliac.reserv.2008

1,114.0

M3

1,442.0
328.0

M3
M3

DECIDIMOS INCREMENTAR EN FORMA PARALELA UNA LINEA DE CONDUCCIN PARA INCREMENTAR EL CAUDAL DE 151 A 186.3, Qd= 35.3 Lt./seg.
POR LO QUE NECESITAMOS INCREMENTAR UN RESERVORIO DE 328 M3 PARA CUBRIR LA NECESIDAD SOLO GENERADA AL 2009
NECESITAMOS CALCULAR LA NECESIDAD PARA EL AO 2031 CONSIDERANDO UN PERIODO DE DISEO INICIAL DE 20 AOS

NDOSE EL Q=186.3 Lit/seg. Cte. como produccin


Q demanda mxima horaria = 201.83

ISTENTE ( 2000) CON PD=15 Aos

CONSUMO DETERMINADO EN BASE A LA PROYECCIN DE VARIACIONES DE CONSUMO Y EL CAUDAL DE DISEO PARA EL AO 2031, USANDO P
Q dis( 2026) =

0.00 litros/seg.

ones de consumo del ao 1999 para las zonas A y B

GASTO

Factor a =

100% demanda.2008

PROM.(2031)
@ 2 horas
% del Qprom.

DE LA LOCAL.

(Lts.)

(Lt./seg.)

(Lts./s.)

VOLUM. DE

HORAS

CONS.RESERVORIO

718,326
1,336,050
1,484,298
827,730
104,094
-619,542
-1,376,712
-1,664,406
-2,119,770
-2,977,542
-3,432,906
-3,049,920
-3,049,920

60.00%
65.00%
70.00%
85.00%
120.00%
115.00%
145.00%
125.00%
110.00%
115.00%
100.00%
90.00%
60.00%

4,917,204

4,917.2

GASTO

VOLUM.

PROM @ 2HR.

AUDAL DE 151 A 186.3, Qd= 35.3 Lt./seg.


O GENERADA AL 2009

(Lts.)

PRODUCC. PROY.

VOLUM. DE

ACUMUL.

ACUM.24Hr.2008

CONS.RESERVORIO

CONSUMO (2008)

(C / 151Lit/seg.)

100% proy.demanda.2015

(Lts.)

(Lts.)

(Lts.)
0

75.00%

0.000

2-4

81.25%

0.000

4-6

87.50%

0.000

6-8

106.25%

0.000

8-10

150.00%

0.000

10-12

143.75%

0.000

12-14

181.25%

0.000

14-16

156.25%

0.000

16-18

137.50%

0.000

18-20

143.75%

0.000

20-22

125.00%

0.000

22-24

112.50%

0.000

0-2

75.00%

0.000

0.00

Reserv(1003.28 l/s.)=

0.00
(Lts./s.)

VOLUMEN

0-2

POR DA

Vol.diseo ampliac.reserv.2008

1.25

LOCALIDAD (Consumo prom.usado en el diseo del proyecto(2000) con variaciones de consumo del ao 1999 para
las zonas A y B Pd=15 aos (2015) )
CUADRO 03.-

0
(Lts.)

0.00

(M3)

Vol.diseo inic.reservorio 2026:

0.00

Se calcula con ste dato (2031) por encontarse cercano al ao del periodo de diseo calculado de 15 aos (2012 + 15 =2027)

EL CAUDAL DE DISEO PARA EL AO 2031, USANDO PD=20 Aos (2012-2031)

RELACIN DIAMETRO - VELOCIDAD ECONMICA

diseo del proyecto(2000) con variaciones de consumo del ao 1999 para

s A y B Pd=15 aos (2015)

(Ref. : NORMAS INOS-VENEZOLANAS)


DIMETRO

M3

x 1.3 =

diseo calculado de 15 aos (2012 + 15 =2027)

0.00 M3

mm.

Pulg.

V mx. (m/seg.)

Q (m3/seg.)
3.05

75

3"

0.70

100

4"

0.75

5.89

150

6"

0.80

14.14

200

8"

0.90

28.27

250

10"

1.00

49.09

300

12"

1.10

77.75

350

14"

1.20

115.45

400

16"

1.25

157.10

450

18"

1.30

206.76

500

20"

1.40

274.90

600

24"

1.60

452.39

750

30"

1.60

729.60

FACULTAD DE INGENIERIA CIVIL Y ARQUITECTURA


ESCUELA ACDEMICA PROFESIONAL DE INGENIERA CIVIL

SOLUCIONARIO DEL EXAMEN DE SUBSANACIN(CICLO 2.2009 (14/03/10) )


RESP.DEL REA HIDRAULICA :
ASIGNATURA

Ing. Civil Clifton Paucar y Montenegro - REG. CIP. 45773


ABASTECIMIENTOS DE AGUA Y ALCANTARILLADO - CIC404

HOJA N 01/09

RPTA A.-

i)

Iniciamos el proceso de estimacin calculando la dotacin promedio CATASTRAL, en funcin al consumo


promedio cuantificado por tipo de EDIFICACIN Y HABITANTE, as como el consumo promedio en Lt./seg.
80,500

Poblacin total localidad ALFA

Habitantes

ZONA A :

Densidad poblacional promedio :

DATOS

TIPO1( unid.)=

1,000

TIPO2( unid.) =

885

TIPO3( unid.) =

500

TIPO4( unid.) =

300

Lit./viv./da

Lit./hab./da

Lit./viv./da

Lit./hab./da

Lit./viv./da

Lit./hab./da

Lit./viv./da

Lit./hab./da

430
345
380

2,230
1,790
2,020

370
295
325

1,400
1,200
1,302

230
188
206

1,250
940
1,120

206
155
195

Mximo
2,600
Mnimo
2,100
Promedio
2,300
TOTAL VIVIENDAS

2,685

Consumo promedio por edificacin/ da =

Poblacin en el Ao 2009:

(RED EXISTENTE)

6.00 habitantes por vivienda

1,000 x 2,300 885 x 2,020 500 x 1,250 300 x 1,118


(1,000865 500300 )

20.00%

80,500 Hab.

Densidad poblacional promedio por edificacin


Dotacin para el AO 2009 en base al consumo por tipo de edificacin

1,890.02 Lit./edif./da

16,100 hab.

6.00 hab

315.00 Lt/hab./da

(x C.R:Includo 17% de prdidas y desperdicios en la red de reciente instalacin)

Dotacin para el AO 2009 en base al consumo por habitante

308.80 Lt/hab./da

(xC.C:Incidencia futura de incremento de consumo por la tendencia de crecimiento comercial)

TOMANDO EL VALOR MAYOR DE DOTACIN TENDREMOS


Consumo promedio instantaneo para la zona A

58.70 Lt./seg.

(RED EXISTENTE)

ZONA B :

Densidad poblacional promedio :

DATOS

TIPO1( unid.)=

2000

TIPO2( unid.) =

1500

TIPO3( unid.) =

800

TIPO4( unid.) =

620

Lit./viv./da

Lit./hab./da

Lit./viv./da

Lit./hab./da

Lit./viv./da

Lit./hab./da

Lit./viv./da

Lit./hab./da

240
190
200

1,810
1,450
1,648

200
160
180

1,750
1,220
1,370

190
135
150

1,400
1,040
1,141

150
115
128

Mximo
2,200
Mnimo
1,750
Promedio
1,850
TOTAL VIVIENDAS

4,920

Consumo promedio por edificacin/ da =


Poblacin en el Ao 2009:

9.00 habitantes por vivienda

2,000 x1,850 1,500 x 1,648 800x 1,370 620x 1,141


( 2,0001,500 800 620 )

55.00%

80,500 Hab.

Densidad poblacional promedio por edificacin


Dotacin para el AO 2009 en base al consumo por tipo de edificacin

1,621.02 Lit./edif./da

44,275 hab.

9.00 hab

180.11 Lt/hab./da

(x C.R:Includo 45% de prdidas y desperdicios en la red de instalacin antigua)

Dotacin para el AO 2009 en base al consumo por habitante

176.70 Lt/hab./da

(xC.C:Incidencia futura de incremento de consumo al mantener la tendencia de crecimiento sostenido de tipo residencial)

TOMANDO EL VALOR MAYOR DE DOTACIN TENDREMOS


Consumo promedio instantaneo para la zona B

92.30 Lt./seg.

(RED EN PROYECCIN)

ZONA C :

Densidad poblacional promedio :

DATOS

TIPO1( unid.)=

1300

TIPO2( unid.) =

1800

TIPO3( unid.) =

1200

TIPO4( unid.) =

730

Lit./viv./da

Lit./hab./da

Lit./viv./da

Lit./hab./da

Lit./viv./da

Lit./hab./da

Lit./viv./da

Lit./hab./da

700
600
650

200
165
180

700
550
600

190
150
165

700
450
500

180
125
132

550
350
350

140
90
100

Mximo
Mnimo
Promedio

4.00 habitantes por vivienda

FACULTAD DE INGENIERIA CIVIL Y ARQUITECTURA


ESCUELA ACDEMICA PROFESIONAL DE INGENIERA CIVIL

SOLUCIONARIO DEL EXAMEN DE SUBSANACIN(CICLO 2.2009 (14/03/10) )


RESP.DEL REA HIDRAULICA :

Ing. Civil Clifton Paucar y Montenegro - REG. CIP. 45773

ABASTECIMIENTOS DE AGUA Y ALCANTARILLADO - CIC404

ASIGNATURA

Consumo promedio por edificacin/ da =

Poblacin en el Ao 2009:

1,300 x 650 1,800 x 600 1,200 x 500 730 x 350


(1,300 1,800 1,200 730 )

25.00%

80,500 Hab.

Densidad poblacional promedio por edificacin


Dotacin para el AO 2009 en base al consumo por tipo de edificacin

HOJA N 02/09

552.78 Lit./edif./da

20,125 hab.

4.00 hab

138.20 Lt/hab./da

(x C.R:Includo 17% de prdidas y desperdicios en la red de reciente instalacin)

Dotacin para el AO 2009 en base al consumo por habitante

151.57 Lt/hab./da

(xC.C:Incidencia futura de incremento de consumo por la tendencia de crecimiento comercial)

TOMANDO EL VALOR MAYOR DE DOTACIN TENDREMOS


Consumo promedio instantaneo para la zona C

35.30 Lt./seg.

Rpta. A y B
ZONA CON SERVICIO DE AGUA

ZONA SIN SERVICIO DE AGUA

CUADRO DE RESPUESTAS A Y B.DEMANDA CATASTRAL PROMEDIO:

Zona "A"

Zona "B"

Zona "C"

Densidad prom.poblac.(hab./edif.)

6.00

9.00

4.00

Poblacin Total ao 2009 ( habitantes )


Dotacin para el ao 2009 (Lt./hab./da)

16,100
315.00

44,275
180.11

20,125
151.57

80,500.00

Consumo prom.ao 2009 ( Lt./seg. )

58.70

92.30

35.30

186.30

Q de diseo del sistema ao 2000(Dato)

Total

DEFICIT./SUPERAV.

123.4%

151.00

Rpta. C
CUADRO DE RESPUESTA PARA EL INCISO C.DEMANDA CATASTRAL POR EDIFICACIN
Includo/PRDIDAS:

Zona "A"

Zona "B"

Zona "C"

6.00

9.00

4.00

Poblacin Total ao 2009 ( habitantes )

16,100

44,275

20,125

Dotacin para el ao 2009 calculada por


edificacin sin prdidas(Lt./hab./da)

315.00

180.11

138.20

Dotacin para el ao 2009 Inc. Prdidas segn datos


suministrados 60%-45%-17% (Lt./hab./da)
Dotacin para el ao 2015 Inc. Prdidas segn datos
proyectados 60%-45%-17% (Lt./hab./da)
Dotacin para el ao 2021 Inc. Prdidas segn datos
proyectados 60%-45%-17% (Lt./hab./da)
Dotacin para el ao 2031 Inc. Prdidas segn datos
proyectados 60%-45%-17% (Lt./hab./da)

504.00

261.16

161.69

504.00

261.16

161.69

504.00

261.16

161.69

504.00

261.16

161.69

Consumo prom.ao 2009 sin / Prdidas


(Lt./seg. )
Consumo prom.ao 2009 includo/ Prdidas
(Lt./seg. )

58.70

92.30

32.19

183.19

93.92

133.83

37.66

265.41

Poblacin proyectada ao 2015

0.00

0.00

0.00

0.00

Consumo prom.ao 2015 sin / Prdidas


(Lt./seg. )
Consumo prom.ao 2015 Includo / Prdidas
(Lt./seg. )

0.00

0.00

0.00

0.00

0.00

0.00

0.00

0.00

Poblacin proyectada ao 2021

0.00

0.00

0.00

0.00

Consumo prom.ao 2021 sin / Prdidas


(Lt./seg. )
Consumo prom.ao 2021 Includo / Prdidas
(Lt./seg. )

0.00

0.00

0.00

0.00

0.00

0.00

0.00

0.00

Poblacin proyectada ao 2031

0.00

0.00

0.00

0.00

Consumo prom.ao 2031 sin / Prdidas


(Lt./seg. )
Consumo prom.ao 2031 Includo / Prdidas
(Lt./seg. )

0.00

0.00

0.00

0.00

0.00

0.00

0.00

0.00

#DIV/0!

PROM.:

#DIV/0!

Densidad prom.poblac.(hab./edif.)

Q de diseo del sistema ao 2000(Dato)

151.00

Total

80,500.00

144.9%

#DIV/0!

#DIV/0!

EL ANLISIS DE LA VARIACIN DE CONSUMO EN EL TIEMPO SE EFECTUAR CON LA DECISIN DE USAR PARA EL CALCULO DE LA
DOTACIN INCREMENTADA , UNA ECUACIN LINEAL CON LOS DATOS (ZONAS URBANAS-DONDE EL INCREMENTO DEL CONSUMO NO
SOLO DEPENDE DEL CRECIMIENTO POBLACIONAL) O EL MODELO MATEMATICO DE CAPEN (ZONAS RURALES-DONDE LA DOTACIN
SE BASA EN EL USO DOMESTICO Y SU VARIACIN DEPENDE DEL INCREMENTO POBLACIONAL), PARA LO CUL ANTES DEBEMOS
ESTIMAR EL QPROMEDIO DE DISEO SEGN LAS CARACTERISTICAS DE CONSUMO DIARIO Y HORARIO, LOS FACTORES DE
MAYORACIN K1 Y K2 Y LA POBLACIN FUTURA

ii)

Efectuamos las estimaciones de las demanda por variaciones de consumo diarias en funcin a los datos
suministrados del diario promedio horario y del diario maximo horario en Lt./seg., a fin de determinar los
valores de K2.

Para determinar el caudal medio en Lt./seg. durante el ao 2009, se proceder a desarrollar el cuadro que contiene las variaciones horarias del
consumo promedio durante el ao, que representa en forma analitica a la curva masa (Produccin y Demanda vrs. Tiempo), teniendo como base que
se consume 151.00 Lt./seg. (2009), y que es la produccin total de la captacin (100%) que es de 151 Lt.seg. con el cul se diseo el reservorio en el
ao 2000, proyecto que entr en funcionamiento el ao 2001 con un Pd=15 aos, debiendo cumplir con eficiencia la demanda hasta el ao 2015,
PERO SE OBSERVA EN EL CUADRO 01 que slo cumple con la demanda del ao 2009 PARA LAS ZONA A Y B, Y QUE DE INCLUIRSE LA
DEMANDA INSATISFECHA EN C QUE NO CUENTA CON SISTEMA, EXISTE UN DEFICIT EN ESTE AO 2009 DE 44.4% .

DATO:

HORAS

Qdisp. Mx.=

1,001 Litros/seg.

LOCALIDAD ( Consumo promedio ao 2009 )

GASTO

CUADRO 02.-

PROM.
@ 2 horas
% del Qprom.

GASTO

VOLUM.(2009)

PRODUCC.(2009)

VOLUM. DE

SIMULACIN

VOLUM. DE

PROM @ 2HR.

VOLUMEN

ACUM.CONSUMO

ACUM.24Hr.

CONS.RESERV.

PRODUCC.(2009)

CONS.RESERV.

(186.30 Lt./seg.)

(186.30 Lt./seg.)

(186.30 Lt./seg.)

(151 Lt./seg.)

(Demanda 186.3)

(186.30 Lt./seg.)

(Demanda 186.30 Lt./s)

(Lt./seg.)

(Lts./s.)

(Lts.)

(Lts.)

(Lts.)

(Lts.)

(Lts.)

(Lts.)

25.00%

46.575

368,874

368,874

1,087,200

718,326

1,341,360

972,486

30.00%

55.890

469,476

838,350

2,174,400

1,336,050

2,682,720

1,844,370

40.00%

74.520

938,952

1,777,302

3,261,600

1,484,298

4,024,080

2,246,778

100.00%

186.300

1,743,768

3,521,070

4,348,800

827,730

5,365,440

1,844,370

160.00%

298.080

1,810,836

5,331,906

5,436,000

104,094

6,706,800

1,374,894

10

110.00%

204.930

1,810,836

7,142,742

6,523,200

-619,542

8,048,160

905,418

12

160.00%

298.080

1,844,370

8,987,112

7,610,400

-1,376,712

9,389,520

402,408

14

115.00%

214.245

1,374,894

10,362,006

8,697,600

-1,664,406

10,730,880

368,874

16

90.00%

167.670

1,542,564

11,904,570

9,784,800

-2,119,770

12,072,240

167,670

18

140.00%

260.820

1,944,972

13,849,542

10,872,000

-2,977,542

13,413,600

-435,942

20

150.00%

279.450

1,542,564

15,392,106

11,959,200

-3,432,906

14,754,960

-637,146

22

80.00%

149.040

704,214

16,096,320

13,046,400

-3,049,920

16,096,320

24

25.00%

46.575

16,096,320

13,046,400

POR DA

2,235.60
(Lts./s.)

16,096,320

16,096.32

(Lts.)

(M3)

186.30

k2prom.09:

Reserv.151 l/s.

16,096,320
4,917,204

Vol.diseo inic.reservorio 95:

Reserv(186.3 l/s.)=

1,441,962

Vol.diseo ampliac.reservor 2008

1.60

VALOR SIN APLICACIN DEL FACTOR DE MAYORACION K1 =1.3


4,917.2
M3
1,442.0
Como se tiene como dato que la persistencia de consumo de los valores mximos diarios es de 10 a 13% se utilizar el cuadro No. 02 para estimar el
valor de K2 y el Volumen de consumo del reservorio principal de ampliacin. Se muestra los graficos de las curvas masa en ambos casos con fines
de comparacin de los consumos.

FACULTAD DE INGENIERIA CIVIL Y ARQUITECTURA


ESCUELA ACDEMICA PROFESIONAL DE INGENIERA CIVIL

SOLUCIONARIO DEL EXAMEN DE SUBSANACIN(CICLO 2.2009 (14/03/10) )


RESP.DEL REA HIDRAULICA :
ASIGNATURA

50,000,000
45,000,000
40,000,000

35,000,000
30,000,000
25,000,000

Ing. Civil Clifton Paucar y Montenegro - REG. CIP. 45773


ABASTECIMIENTOS DE AGUA Y ALCANTARILLADO - CIC404

HOJA N 03/09

25,000,000
20,000,000

15,000,000
10,000,000
5,000,000
0

10

12

14

16

18

20

22

SIMULACIN PRODUCC.(2009) (54.45 Lt./seg.) (Lts.)


PRODUCC.(2009) ACUM.24Hr. (9.5 Lt./seg.) (Lts.)

SIMULACIN PRODUCC.(2009) (54.45 Lt./seg.) (Lts.)


PRODUCC.(2009) ACUM.24Hr. (54.45Lt./seg.) (Lts.)

VOLUM. ACUMUL. Cons.da mx. (Lts.)

HORAS

GASTO
CUADRO 03.PROM.
GASTO
@ 2 horas
PROM @ 2HR.
% del Qprom.
(186.30 Lt./seg.)
(Lt./seg.)

(Lts./s.)

LOCALIDAD (Consumo del da mximo ao 2009 )


VOLUM.

PRODUCC.(2009)

VOLUM. DE

VOLUMEN

ACUMUL.

ACUM.24Hr.

CONSUM.RES. (100%)

(186.30 Lt./seg.)

Cons.da mx.

(186.3 Lt./seg.)

Cons.da mx.

(Lts.)

(Lts.)

(Lts.)

(Lts.)

30.00%

55.89

435,960

435,960

1,087,200

651,240

35.00%

65.21

536,580

972,540

2,174,400

1,201,860
1,283,004

45.00%

83.84

1,006,056

1,978,596

3,261,600

105.00%

195.62

1,810,872

3,789,468

4,348,800

559,332

165.00%

307.40

1,911,456

5,700,924

5,436,000

-264,924

10

120.00%

223.56

2,146,176

7,847,100

6,523,200

-1,323,900

12

200.00%

372.60

2,112,660

9,959,760

7,610,400

-2,349,360

14

115.00%

214.25

1,408,464

11,368,224

8,697,600

-2,670,624

16

95.00%

176.99

1,609,668

12,977,892

9,784,800

-3,193,092

18

145.00%

270.14

2,045,592

15,023,484

10,872,000

-4,151,484

20

160.00%

298.08

1,643,184

16,666,668

11,959,200

-4,707,468

22

85.00%

158.36

771,300

17,437,968

13,046,400

-4,391,568

24

30.00%

55.89

17,437,968

13,046,400

-4,391,568

17,437.97

Reserv(186.3 l./s.)=

5,990,472

POR DA

2,421.94
(Lts./s.)

Rpta. D:

k1=

1.30

17,437,968
(Lts.)

(M3)

Vol.diseo ampliac.reservor 2008 por cons.mx.horar.

201.83

k2dis.(2008):

2.00 (Coef.da de mx.consumo)

(Reglam.)

k2dis.09:

1.60 (Consumo Promedio)

5,990.00

M3

Yi 15.77 e

0.00005

Xi
CURVA DE CRECIMIENTO POBLACIONAL ZONA B

20,000

18,000

POBLACIN

16,000

14,000

12,000

10,000

8,000

6,000

4,000

2,000

0
2004 2009 2014 2019 2024 2029 2034 2039 2044 2049 2054 2059 2064 2069 2074

AO

POBLACIN

Yi 25.66 0.00015 X i

TASAS CREC. POB.

CURVA DE CRECIMIENTO POBLACIONAL ZONA A


1

POBLACIN

0
1

AO

Yi 77.17 e

0.00004

Xi

CURVA DE CRECIMIENTO POBLACIONAL ZONA A

CURVA DE CRECIMIENTO POBLACIONAL ZONA A


60,000

50,000

POBLACIN

40,000

30,000

20,000

10,000

0
2004

2009

2014

2019

2024

2029

AO

POBLACIN

TASAS CREC. POB.

2034

2039

2044

1 Pto.
1 Pto.

1 Pto.

1 Pto.

SE USARA LA VARIACION LINEAL POR


QUE LA DIFERENCIA ES SIGNIFICATIVA
CON LA PROYECCION USANDO EL
MODELO MATEMATICO DE CAPEN

Q diseo = 1.3 X

186.30

242.19

VOLUM.

PRODUCC.PROY.x1.3

VOLUM. DE

Lit./seg.

ACUMUL. X 1.3
CONSUMO (2009)

ACUM.24Hr.2009
(C /242.19 Lit/seg.)

(Lts.)

(Lts.)

Caudal de diseo de la tubera de entrada del Reservorio

CONS.RESERVORIO

Usando la tabla que seala velocidades mximas y gastos mximos admisibles para
cada dimetro, propuesta por las normas Venezolanas (INOS-Ref.3)

Mayorado c/ K1=1,3

(Lts.)

=
479,536

1,413,360

933,824

1,089,855

2,826,720

1,736,865

2,310,493

4,240,080

1,929,587

4,577,391

5,653,440

1,076,049

6,931,478

7,066,800

135,322

9,285,565

8,480,160

-805,405

11,683,246

9,893,520

-1,789,726

13,470,608

11,306,880

-2,163,728

15,475,941

12,720,240

-2,755,701

18,004,405

14,133,600

-3,870,805

20,009,738

15,546,960

-4,462,778

20,925,216

16,960,320

-3,964,896

20,925,216

16,960,320

4,917 X1.3

20"
REQUERIMIENTO DE DISEO (2009)
186.30 x 1.3 =
242.19 l/s
20"

ANTERIOR DISEO (2000)

151.00 l/s
16"

-3,964,896

6,392,365
6,392.4 M3

6,392.4

1,442.0 X 1.3 =

1,875 M3

Diferencia generada por la produccin, que se entiende fsicamente que si no se cambia el dimetro de la tubera de acceso al Reservorio se tendra que
construir un RESERVORIO de (4,917.2-1,442.0=3,475.20 m3), considerando que en el ao 2000 se uso los mismos % de variaciones de consumo del 2009; si se
considera el cambio de tubera de acceso para transportar 186.30 Lt./s. solo se necesitara u Reservorio de 1, 442 m3 que deber contrastarse con el volumen de
consumo del reservorio construdo ( Evaluacin de costos y ventajas tcnicas)

-3,475.2 M3
DEFICIT DE SERVICIO EN EL AO 2009 AL PRODUCIR 151 Lit./ seg. PARA CUBRIR UNA DEMANDA DE 186.30 Lit./s. NETA DE
CONSUMO POR EDIFICACIN SIN CONSIDERAR PERDIDAS+CONS.PBLICO+OPERAC.YMANTEN.VARIAC. DE DOTACIN
CON EL TIEMPO.

DECIDIMOS INCREMENTAR E
POR LO QUE NECESITAMOS
NECESITAMOS CALCULAR LA

M3

2 Ptos.

VARIA NOTORIAMENTE AL INCREMENTARSE LOS % DE VARIACIONDE CONSUMO MANTENIENDOSE EL Q=186.3 Lit/seg. Cte. como
1,442.0
5,990.00
Q demanda mxima horaria = 20
<
EL VOLUMEN DEL RESERVORIO EN EL 2031 SERA:
1,442.0

1.3

1,874.6 M3

CONSUMO DETERMINADO EN EL EXPEDIENTE DEL PROYECTO PARA DETERMINAR EL VOLUMEN DEL RESERVORIO EXISTENTE ( 2000) CON PD=15 Aos

GASTO
HORAS

CUADRO 03.-

PROM.(1999)
@ 2 horas
% del Qprom.

DE LA LOCAL.

(Lt./seg.)

(Lts./s.)

LOCALIDAD (Consumo prom.usado en el diseo del proyecto(2000) con variaciones de consumo del ao 1999 para las zonas A y B
Pd=15 aos (2015))

GASTO
PROM @ 2HR.

VOLUM.
VOLUMEN

(Lts.)

PRODUCC. PROY.

VOLUM. DE

CONSUMO REAL

ACUMUL.

ACUM.24Hr.2015

CONS.RESERVORIO

ACUM.24Hr.2009

CONSUMO (2015)

(C / 151Lit/seg.)

100% proy.demanda.2015

(C / 186.30 Lit/seg.)

(Lts.)

(Lts.)

(Lts.)

(Lts.)

0-2

5.00%

7.550

108,720

108,720

1,087,200

978,480

2-4

15.00%

22.650

217,440

326,160

2,174,400

1,848,240

368,874
838,350

4-6

25.00%

37.750

597,960

924,120

3,261,600

2,337,480

1,777,302

6-8

85.00%

128.350

1,304,640

2,228,760

4,348,800

2,120,040

3,521,070

8-10

155.00%

234.050

1,522,080

3,750,840

5,436,000

1,685,160

5,331,906

10-12

125.00%

188.750

2,147,220

5,898,060

6,523,200

625,140

7,142,742

12-14

270.00%

407.700

2,283,120

8,181,180

7,610,400

-570,780

8,987,112

14-16

150.00%

226.500

1,386,180

9,567,360

8,697,600

-869,760

10,362,006

16-18

105.00%

158.550

1,359,000

10,926,360

9,784,800

-1,141,560

11,904,570

18-20

145.00%

218.950

1,168,740

12,095,100

10,872,000

-1,223,100

13,849,542

20-22

70.00%

105.700

652,320

12,747,420

11,959,200

-788,220

15,392,106

22-24

50.00%

75.500

298,980

13,046,400

13,046,400

16,096,320

0-2

5.00%

7.550

13,046,400

13,046,400

13,046.40

Reserv(343.68 l/s.)=

1,114,380

Reserv(343.68 l/s.)=

POR DA

1,812.00
(Lts./s.)

13,046,400
(Lts.)

(M3)

151.00

Volumen de consumo del Reservorio si se utiliza los % de variacin del 2009


DIFERENCIA

16,096,320

Vol.diseo inic.reservorio 95:

Vol.diseo ampliac.reserv.2008

1,114.0

M3

1,442.0
328.0

M3
M3

DECIDIMOS INCREMENTAR EN FORMA PARALELA UNA LINEA DE CONDUCCIN PARA INCREMENTAR EL CAUDAL DE 151 A 186.3, Qd= 35.3 Lt./seg.
POR LO QUE NECESITAMOS INCREMENTAR UN RESERVORIO DE 328 M3 PARA CUBRIR LA NECESIDAD SOLO GENERADA AL 2009
NECESITAMOS CALCULAR LA NECESIDAD PARA EL AO 2031 CONSIDERANDO UN PERIODO DE DISEO INICIAL DE 20 AOS

NDOSE EL Q=186.3 Lit/seg. Cte. como produccin


Q demanda mxima horaria = 201.83

ISTENTE ( 2000) CON PD=15 Aos

CONSUMO DETERMINADO EN BASE A LA PROYECCIN DE VARIACIONES DE CONSUMO Y EL CAUDAL DE DISEO PARA EL AO 2031, USANDO P
Q dis( 2026) =

0.00 litros/seg.

ones de consumo del ao 1999 para las zonas A y B

GASTO

Factor a =

100% demanda.2008

PROM.(2031)
@ 2 horas
% del Qprom.

DE LA LOCAL.

(Lts.)

(Lt./seg.)

(Lts./s.)

VOLUM. DE

HORAS

CONS.RESERVORIO

718,326
1,336,050
1,484,298
827,730
104,094
-619,542
-1,376,712
-1,664,406
-2,119,770
-2,977,542
-3,432,906
-3,049,920
-3,049,920

60.00%
65.00%
70.00%
85.00%
120.00%
115.00%
145.00%
125.00%
110.00%
115.00%
100.00%
90.00%
60.00%

4,917,204

4,917.2

GASTO

VOLUM.

PROM @ 2HR.

AUDAL DE 151 A 186.3, Qd= 35.3 Lt./seg.


O GENERADA AL 2009

(Lts.)

PRODUCC. PROY.

VOLUM. DE

ACUMUL.

ACUM.24Hr.2008

CONS.RESERVORIO

CONSUMO (2008)

(C / 151Lit/seg.)

100% proy.demanda.2015

(Lts.)

(Lts.)

(Lts.)
0

54.00%

0.000

2-4

58.50%

0.000

4-6

63.00%

0.000

6-8

76.50%

0.000

8-10

108.00%

0.000

10-12

103.50%

0.000

12-14

130.50%

0.000

14-16

112.50%

0.000

16-18

99.00%

0.000

18-20

103.50%

0.000

20-22

90.00%

0.000

22-24

81.00%

0.000

0-2

54.00%

0.000

0.00

Reserv(1003.28 l/s.)=

0.00
(Lts./s.)

VOLUMEN

0-2

POR DA

Vol.diseo ampliac.reserv.2008

0.9

LOCALIDAD (Consumo prom.usado en el diseo del proyecto(2000) con variaciones de consumo del ao 1999 para
las zonas A y B Pd=15 aos (2015) )
CUADRO 03.-

0
(Lts.)

0.00

(M3)

Vol.diseo inic.reservorio 2026:

0.00

Se calcula con ste dato (2031) por encontarse cercano al ao del periodo de diseo calculado de 15 aos (2012 + 15 =2027)

EL CAUDAL DE DISEO PARA EL AO 2031, USANDO PD=20 Aos (2012-2031)

RELACIN DIAMETRO - VELOCIDAD ECONMICA

diseo del proyecto(2000) con variaciones de consumo del ao 1999 para

s A y B Pd=15 aos (2015)

(Ref. : NORMAS INOS-VENEZOLANAS)


DIMETRO

M3

x 1.3 =

diseo calculado de 15 aos (2012 + 15 =2027)

0.00 M3

mm.

Pulg.

V mx. (m/seg.)

Q (m3/seg.)
3.05

75

3"

0.70

100

4"

0.75

5.89

150

6"

0.80

14.14

200

8"

0.90

28.27

250

10"

1.00

49.09

300

12"

1.10

77.75

350

14"

1.20

115.45

400

16"

1.25

157.10

450

18"

1.30

206.76

500

20"

1.40

274.90

600

24"

1.60

452.39

750

30"

1.60

729.60

FACULTAD DE INGENIERIA CIVIL Y ARQUITECTURA


ESCUELA ACDEMICA PROFESIONAL DE INGENIERA CIVIL

SOLUCIONARIO DEL EXAMEN DE SUBSANACIN(CICLO 2.2009 (14/03/10) )


RESP.DEL REA HIDRAULICA :
ASIGNATURA

Ing. Civil Clifton Paucar y Montenegro - REG. CIP. 45773


ABASTECIMIENTOS DE AGUA Y ALCANTARILLADO - CIC404

HOJA N 01/09

RPTA A.-

i)

Iniciamos el proceso de estimacin calculando la dotacin promedio CATASTRAL, en funcin al consumo


promedio cuantificado por tipo de EDIFICACIN Y HABITANTE, as como el consumo promedio en Lt./seg.
80,500

Poblacin total localidad ALFA

Habitantes

ZONA A :

Densidad poblacional promedio :

DATOS

TIPO1( unid.)=

1,000

TIPO2( unid.) =

885

TIPO3( unid.) =

500

TIPO4( unid.) =

300

Lit./viv./da

Lit./hab./da

Lit./viv./da

Lit./hab./da

Lit./viv./da

Lit./hab./da

Lit./viv./da

Lit./hab./da

430
345
380

2,230
1,790
2,020

370
295
325

1,400
1,200
1,302

230
188
206

1,250
940
1,120

206
155
195

Mximo
2,600
Mnimo
2,100
Promedio
2,300
TOTAL VIVIENDAS

2,685

Consumo promedio por edificacin/ da =

Poblacin en el Ao 2009:

(RED EXISTENTE)

6.00 habitantes por vivienda

1,000 x 2,300 885 x 2,020 500 x 1,250 300 x 1,118


(1,000865 500300 )

20.00%

80,500 Hab.

Densidad poblacional promedio por edificacin


Dotacin para el AO 2009 en base al consumo por tipo de edificacin

1,890.02 Lit./edif./da

16,100 hab.

6.00 hab

315.00 Lt/hab./da

(x C.R:Includo 17% de prdidas y desperdicios en la red de reciente instalacin)

Dotacin para el AO 2009 en base al consumo por habitante

308.80 Lt/hab./da

(xC.C:Incidencia futura de incremento de consumo por la tendencia de crecimiento comercial)

TOMANDO EL VALOR MAYOR DE DOTACIN TENDREMOS


Consumo promedio instantaneo para la zona A

58.70 Lt./seg.

(RED EXISTENTE)

ZONA B :

Densidad poblacional promedio :

DATOS

TIPO1( unid.)=

2000

TIPO2( unid.) =

1500

TIPO3( unid.) =

800

TIPO4( unid.) =

620

Lit./viv./da

Lit./hab./da

Lit./viv./da

Lit./hab./da

Lit./viv./da

Lit./hab./da

Lit./viv./da

Lit./hab./da

240
190
200

1,810
1,450
1,648

200
160
180

1,750
1,220
1,370

190
135
150

1,400
1,040
1,141

150
115
128

Mximo
2,200
Mnimo
1,750
Promedio
1,850
TOTAL VIVIENDAS

4,920

Consumo promedio por edificacin/ da =


Poblacin en el Ao 2009:

9.00 habitantes por vivienda

2,000 x1,850 1,500 x 1,648 800x 1,370 620x 1,141


( 2,0001,500 800 620 )

55.00%

80,500 Hab.

Densidad poblacional promedio por edificacin


Dotacin para el AO 2009 en base al consumo por tipo de edificacin

1,621.02 Lit./edif./da

44,275 hab.

9.00 hab

180.11 Lt/hab./da

(x C.R:Includo 45% de prdidas y desperdicios en la red de instalacin antigua)

Dotacin para el AO 2009 en base al consumo por habitante

176.70 Lt/hab./da

(xC.C:Incidencia futura de incremento de consumo al mantener la tendencia de crecimiento sostenido de tipo residencial)

TOMANDO EL VALOR MAYOR DE DOTACIN TENDREMOS


Consumo promedio instantaneo para la zona B

92.30 Lt./seg.

(RED EN PROYECCIN)

ZONA C :

Densidad poblacional promedio :

DATOS

TIPO1( unid.)=

1300

TIPO2( unid.) =

1800

TIPO3( unid.) =

1200

TIPO4( unid.) =

730

Lit./viv./da

Lit./hab./da

Lit./viv./da

Lit./hab./da

Lit./viv./da

Lit./hab./da

Lit./viv./da

Lit./hab./da

700
600
650

200
165
180

700
550
600

190
150
165

700
450
500

180
125
132

550
350
350

140
90
100

Mximo
Mnimo
Promedio

4.00 habitantes por vivienda

FACULTAD DE INGENIERIA CIVIL Y ARQUITECTURA


ESCUELA ACDEMICA PROFESIONAL DE INGENIERA CIVIL

SOLUCIONARIO DEL EXAMEN DE SUBSANACIN(CICLO 2.2009 (14/03/10) )


RESP.DEL REA HIDRAULICA :

Ing. Civil Clifton Paucar y Montenegro - REG. CIP. 45773

ABASTECIMIENTOS DE AGUA Y ALCANTARILLADO - CIC404

ASIGNATURA

Consumo promedio por edificacin/ da =

Poblacin en el Ao 2009:

1,300 x 650 1,800 x 600 1,200 x 500 730 x 350


(1,300 1,800 1,200 730 )

25.00%

80,500 Hab.

Densidad poblacional promedio por edificacin


Dotacin para el AO 2009 en base al consumo por tipo de edificacin

HOJA N 02/09

552.78 Lit./edif./da

20,125 hab.

4.00 hab

138.20 Lt/hab./da

(x C.R:Includo 17% de prdidas y desperdicios en la red de reciente instalacin)

Dotacin para el AO 2009 en base al consumo por habitante

151.57 Lt/hab./da

(xC.C:Incidencia futura de incremento de consumo por la tendencia de crecimiento comercial)

TOMANDO EL VALOR MAYOR DE DOTACIN TENDREMOS


Consumo promedio instantaneo para la zona C

35.30 Lt./seg.

Rpta. A y B
ZONA CON SERVICIO DE AGUA

ZONA SIN SERVICIO DE AGUA

CUADRO DE RESPUESTAS A Y B.DEMANDA CATASTRAL PROMEDIO:

Zona "A"

Zona "B"

Zona "C"

Densidad prom.poblac.(hab./edif.)

6.00

9.00

4.00

Poblacin Total ao 2009 ( habitantes )


Dotacin para el ao 2009 (Lt./hab./da)

16,100
315.00

44,275
180.11

20,125
151.57

80,500.00

Consumo prom.ao 2009 ( Lt./seg. )

58.70

92.30

35.30

186.30

Q de diseo del sistema ao 2000(Dato)

Total

DEFICIT./SUPERAV.

123.4%

151.00

Rpta. C
CUADRO DE RESPUESTA PARA EL INCISO C.DEMANDA CATASTRAL POR EDIFICACIN
Includo/PRDIDAS:

Zona "A"

Zona "B"

Zona "C"

6.00

9.00

4.00

Poblacin Total ao 2009 ( habitantes )

16,100

44,275

20,125

Dotacin para el ao 2009 calculada por


edificacin sin prdidas(Lt./hab./da)

315.00

180.11

138.20

Dotacin para el ao 2009 Inc. Prdidas segn datos


suministrados 60%-45%-17% (Lt./hab./da)
Dotacin para el ao 2015 Inc. Prdidas segn datos
proyectados 60%-45%-17% (Lt./hab./da)
Dotacin para el ao 2021 Inc. Prdidas segn datos
proyectados 60%-45%-17% (Lt./hab./da)
Dotacin para el ao 2031 Inc. Prdidas segn datos
proyectados 60%-45%-17% (Lt./hab./da)

504.00

261.16

161.69

504.00

261.16

161.69

504.00

261.16

161.69

504.00

261.16

161.69

Consumo prom.ao 2009 sin / Prdidas


(Lt./seg. )
Consumo prom.ao 2009 includo/ Prdidas
(Lt./seg. )

58.70

92.30

32.19

183.19

93.92

133.83

37.66

265.41

Poblacin proyectada ao 2015

0.00

0.00

0.00

0.00

Consumo prom.ao 2015 sin / Prdidas


(Lt./seg. )
Consumo prom.ao 2015 Includo / Prdidas
(Lt./seg. )

0.00

0.00

0.00

0.00

0.00

0.00

0.00

0.00

Poblacin proyectada ao 2021

0.00

0.00

0.00

0.00

Consumo prom.ao 2021 sin / Prdidas


(Lt./seg. )
Consumo prom.ao 2021 Includo / Prdidas
(Lt./seg. )

0.00

0.00

0.00

0.00

0.00

0.00

0.00

0.00

Poblacin proyectada ao 2031

0.00

0.00

0.00

0.00

Consumo prom.ao 2031 sin / Prdidas


(Lt./seg. )
Consumo prom.ao 2031 Includo / Prdidas
(Lt./seg. )

0.00

0.00

0.00

0.00

0.00

0.00

0.00

0.00

#DIV/0!

PROM.:

#DIV/0!

Densidad prom.poblac.(hab./edif.)

Q de diseo del sistema ao 2000(Dato)

151.00

Total

80,500.00

144.9%

#DIV/0!

#DIV/0!

EL ANLISIS DE LA VARIACIN DE CONSUMO EN EL TIEMPO SE EFECTUAR CON LA DECISIN DE USAR PARA EL CALCULO DE LA
DOTACIN INCREMENTADA , UNA ECUACIN LINEAL CON LOS DATOS (ZONAS URBANAS-DONDE EL INCREMENTO DEL CONSUMO NO
SOLO DEPENDE DEL CRECIMIENTO POBLACIONAL) O EL MODELO MATEMATICO DE CAPEN (ZONAS RURALES-DONDE LA DOTACIN
SE BASA EN EL USO DOMESTICO Y SU VARIACIN DEPENDE DEL INCREMENTO POBLACIONAL), PARA LO CUL ANTES DEBEMOS
ESTIMAR EL QPROMEDIO DE DISEO SEGN LAS CARACTERISTICAS DE CONSUMO DIARIO Y HORARIO, LOS FACTORES DE
MAYORACIN K1 Y K2 Y LA POBLACIN FUTURA

ii)

Efectuamos las estimaciones de las demanda por variaciones de consumo diarias en funcin a los datos
suministrados del diario promedio horario y del diario maximo horario en Lt./seg., a fin de determinar los
valores de K2.

Para determinar el caudal medio en Lt./seg. durante el ao 2009, se proceder a desarrollar el cuadro que contiene las variaciones horarias del
consumo promedio durante el ao, que representa en forma analitica a la curva masa (Produccin y Demanda vrs. Tiempo), teniendo como base que
se consume 151.00 Lt./seg. (2009), y que es la produccin total de la captacin (100%) que es de 151 Lt.seg. con el cul se diseo el reservorio en el
ao 2000, proyecto que entr en funcionamiento el ao 2001 con un Pd=15 aos, debiendo cumplir con eficiencia la demanda hasta el ao 2015,
PERO SE OBSERVA EN EL CUADRO 01 que slo cumple con la demanda del ao 2009 PARA LAS ZONA A Y B, Y QUE DE INCLUIRSE LA
DEMANDA INSATISFECHA EN C QUE NO CUENTA CON SISTEMA, EXISTE UN DEFICIT EN ESTE AO 2009 DE 44.4% .

DATO:

HORAS

Qdisp. Mx.=

1,001 Litros/seg.

LOCALIDAD ( Consumo promedio ao 2009 )

GASTO

CUADRO 02.-

PROM.
@ 2 horas
% del Qprom.

GASTO

VOLUM.(2009)

PRODUCC.(2009)

VOLUM. DE

SIMULACIN

VOLUM. DE

PROM @ 2HR.

VOLUMEN

ACUM.CONSUMO

ACUM.24Hr.

CONS.RESERV.

PRODUCC.(2009)

CONS.RESERV.

(186.30 Lt./seg.)

(186.30 Lt./seg.)

(186.30 Lt./seg.)

(151 Lt./seg.)

(Demanda 186.3)

(186.30 Lt./seg.)

(Demanda 186.30 Lt./s)

(Lt./seg.)

(Lts./s.)

(Lts.)

(Lts.)

(Lts.)

(Lts.)

(Lts.)

(Lts.)

25.00%

46.575

368,874

368,874

1,087,200

718,326

1,341,360

972,486

30.00%

55.890

469,476

838,350

2,174,400

1,336,050

2,682,720

1,844,370

40.00%

74.520

938,952

1,777,302

3,261,600

1,484,298

4,024,080

2,246,778

100.00%

186.300

1,743,768

3,521,070

4,348,800

827,730

5,365,440

1,844,370

160.00%

298.080

1,810,836

5,331,906

5,436,000

104,094

6,706,800

1,374,894

10

110.00%

204.930

1,810,836

7,142,742

6,523,200

-619,542

8,048,160

905,418

12

160.00%

298.080

1,844,370

8,987,112

7,610,400

-1,376,712

9,389,520

402,408

14

115.00%

214.245

1,374,894

10,362,006

8,697,600

-1,664,406

10,730,880

368,874

16

90.00%

167.670

1,542,564

11,904,570

9,784,800

-2,119,770

12,072,240

167,670

18

140.00%

260.820

1,944,972

13,849,542

10,872,000

-2,977,542

13,413,600

-435,942

20

150.00%

279.450

1,542,564

15,392,106

11,959,200

-3,432,906

14,754,960

-637,146

22

80.00%

149.040

704,214

16,096,320

13,046,400

-3,049,920

16,096,320

24

25.00%

46.575

16,096,320

13,046,400

POR DA

2,235.60
(Lts./s.)

16,096,320

16,096.32

(Lts.)

(M3)

186.30

k2prom.09:

Reserv.151 l/s.

16,096,320
4,917,204

Vol.diseo inic.reservorio 95:

Reserv(186.3 l/s.)=

1,441,962

Vol.diseo ampliac.reservor 2008

1.60

VALOR SIN APLICACIN DEL FACTOR DE MAYORACION K1 =1.3


4,917.2
M3
1,442.0
Como se tiene como dato que la persistencia de consumo de los valores mximos diarios es de 10 a 13% se utilizar el cuadro No. 02 para estimar el
valor de K2 y el Volumen de consumo del reservorio principal de ampliacin. Se muestra los graficos de las curvas masa en ambos casos con fines
de comparacin de los consumos.

FACULTAD DE INGENIERIA CIVIL Y ARQUITECTURA


ESCUELA ACDEMICA PROFESIONAL DE INGENIERA CIVIL

SOLUCIONARIO DEL EXAMEN DE SUBSANACIN(CICLO 2.2009 (14/03/10) )


RESP.DEL REA HIDRAULICA :
ASIGNATURA

50,000,000
45,000,000
40,000,000

35,000,000
30,000,000
25,000,000

Ing. Civil Clifton Paucar y Montenegro - REG. CIP. 45773


ABASTECIMIENTOS DE AGUA Y ALCANTARILLADO - CIC404

HOJA N 03/09

25,000,000
20,000,000

15,000,000
10,000,000
5,000,000
0

10

12

14

16

18

20

22

SIMULACIN PRODUCC.(2009) (54.45 Lt./seg.) (Lts.)


PRODUCC.(2009) ACUM.24Hr. (9.5 Lt./seg.) (Lts.)

SIMULACIN PRODUCC.(2009) (54.45 Lt./seg.) (Lts.)


PRODUCC.(2009) ACUM.24Hr. (54.45Lt./seg.) (Lts.)

VOLUM. ACUMUL. Cons.da mx. (Lts.)

HORAS

GASTO
CUADRO 03.PROM.
GASTO
@ 2 horas
PROM @ 2HR.
% del Qprom.
(186.30 Lt./seg.)
(Lt./seg.)

(Lts./s.)

LOCALIDAD (Consumo del da mximo ao 2009 )


VOLUM.

PRODUCC.(2009)

VOLUM. DE

VOLUMEN

ACUMUL.

ACUM.24Hr.

CONSUM.RES. (100%)

(186.30 Lt./seg.)

Cons.da mx.

(186.3 Lt./seg.)

Cons.da mx.

(Lts.)

(Lts.)

(Lts.)

(Lts.)

30.00%

55.89

435,960

435,960

1,087,200

651,240

35.00%

65.21

536,580

972,540

2,174,400

1,201,860
1,283,004

45.00%

83.84

1,006,056

1,978,596

3,261,600

105.00%

195.62

1,810,872

3,789,468

4,348,800

559,332

165.00%

307.40

1,911,456

5,700,924

5,436,000

-264,924

10

120.00%

223.56

2,146,176

7,847,100

6,523,200

-1,323,900

12

200.00%

372.60

2,112,660

9,959,760

7,610,400

-2,349,360

14

115.00%

214.25

1,408,464

11,368,224

8,697,600

-2,670,624

16

95.00%

176.99

1,609,668

12,977,892

9,784,800

-3,193,092

18

145.00%

270.14

2,045,592

15,023,484

10,872,000

-4,151,484

20

160.00%

298.08

1,643,184

16,666,668

11,959,200

-4,707,468

22

85.00%

158.36

771,300

17,437,968

13,046,400

-4,391,568

24

30.00%

55.89

17,437,968

13,046,400

-4,391,568

17,437.97

Reserv(186.3 l./s.)=

5,990,472

POR DA

2,421.94
(Lts./s.)

Rpta. D:

k1=

1.30

17,437,968
(Lts.)

(M3)

Vol.diseo ampliac.reservor 2008 por cons.mx.horar.

201.83

k2dis.(2008):

2.00 (Coef.da de mx.consumo)

(Reglam.)

k2dis.09:

1.60 (Consumo Promedio)

5,990.00

M3

Yi 15.77 e

0.00005

Xi
CURVA DE CRECIMIENTO POBLACIONAL ZONA B

20,000

18,000

POBLACIN

16,000

14,000

12,000

10,000

8,000

6,000

4,000

2,000

0
2004 2009 2014 2019 2024 2029 2034 2039 2044 2049 2054 2059 2064 2069 2074

AO

POBLACIN

Yi 25.66 0.00015 X i

TASAS CREC. POB.

CURVA DE CRECIMIENTO POBLACIONAL ZONA A


1

POBLACIN

0
1

AO

Yi 77.17 e

0.00004

Xi

CURVA DE CRECIMIENTO POBLACIONAL ZONA A

CURVA DE CRECIMIENTO POBLACIONAL ZONA A


60,000

50,000

POBLACIN

40,000

30,000

20,000

10,000

0
2004

2009

2014

2019

2024

2029

AO

POBLACIN

TASAS CREC. POB.

2034

2039

2044

1 Pto.
1 Pto.

1 Pto.

1 Pto.

SE USARA LA VARIACION LINEAL POR


QUE LA DIFERENCIA ES SIGNIFICATIVA
CON LA PROYECCION USANDO EL
MODELO MATEMATICO DE CAPEN

Q diseo = 1.3 X

186.30

242.19

VOLUM.

PRODUCC.PROY.x1.3

VOLUM. DE

Lit./seg.

ACUMUL. X 1.3
CONSUMO (2009)

ACUM.24Hr.2009
(C /242.19 Lit/seg.)

(Lts.)

(Lts.)

Caudal de diseo de la tubera de entrada del Reservorio

CONS.RESERVORIO

Usando la tabla que seala velocidades mximas y gastos mximos admisibles para
cada dimetro, propuesta por las normas Venezolanas (INOS-Ref.3)

Mayorado c/ K1=1,3

(Lts.)

=
479,536

1,413,360

933,824

1,089,855

2,826,720

1,736,865

2,310,493

4,240,080

1,929,587

4,577,391

5,653,440

1,076,049

6,931,478

7,066,800

135,322

9,285,565

8,480,160

-805,405

11,683,246

9,893,520

-1,789,726

13,470,608

11,306,880

-2,163,728

15,475,941

12,720,240

-2,755,701

18,004,405

14,133,600

-3,870,805

20,009,738

15,546,960

-4,462,778

20,925,216

16,960,320

-3,964,896

20,925,216

16,960,320

4,917 X1.3

20"
REQUERIMIENTO DE DISEO (2009)
186.30 x 1.3 =
242.19 l/s
20"

ANTERIOR DISEO (2000)

151.00 l/s
16"

-3,964,896

6,392,365
6,392.4 M3

6,392.4

1,442.0 X 1.3 =

1,875 M3

Diferencia generada por la produccin, que se entiende fsicamente que si no se cambia el dimetro de la tubera de acceso al Reservorio se tendra que
construir un RESERVORIO de (4,917.2-1,442.0=3,475.20 m3), considerando que en el ao 2000 se uso los mismos % de variaciones de consumo del 2009; si se
considera el cambio de tubera de acceso para transportar 186.30 Lt./s. solo se necesitara u Reservorio de 1, 442 m3 que deber contrastarse con el volumen de
consumo del reservorio construdo ( Evaluacin de costos y ventajas tcnicas)

-3,475.2 M3
DEFICIT DE SERVICIO EN EL AO 2009 AL PRODUCIR 151 Lit./ seg. PARA CUBRIR UNA DEMANDA DE 186.30 Lit./s. NETA DE
CONSUMO POR EDIFICACIN SIN CONSIDERAR PERDIDAS+CONS.PBLICO+OPERAC.YMANTEN.VARIAC. DE DOTACIN
CON EL TIEMPO.

DECIDIMOS INCREMENTAR E
POR LO QUE NECESITAMOS
NECESITAMOS CALCULAR LA

M3

2 Ptos.

VARIA NOTORIAMENTE AL INCREMENTARSE LOS % DE VARIACIONDE CONSUMO MANTENIENDOSE EL Q=186.3 Lit/seg. Cte. como
1,442.0
5,990.00
Q demanda mxima horaria = 20
<
EL VOLUMEN DEL RESERVORIO EN EL 2031 SERA:
1,442.0

1.3

1,874.6 M3

CONSUMO DETERMINADO EN EL EXPEDIENTE DEL PROYECTO PARA DETERMINAR EL VOLUMEN DEL RESERVORIO EXISTENTE ( 2000) CON PD=15 Aos

GASTO
HORAS

CUADRO 03.-

PROM.(1999)
@ 2 horas
% del Qprom.

DE LA LOCAL.

(Lt./seg.)

(Lts./s.)

LOCALIDAD (Consumo prom.usado en el diseo del proyecto(2000) con variaciones de consumo del ao 1999 para las zonas A y B
Pd=15 aos (2015))

GASTO
PROM @ 2HR.

VOLUM.
VOLUMEN

(Lts.)

PRODUCC. PROY.

VOLUM. DE

CONSUMO REAL

ACUMUL.

ACUM.24Hr.2015

CONS.RESERVORIO

ACUM.24Hr.2009

CONSUMO (2015)

(C / 151Lit/seg.)

100% proy.demanda.2015

(C / 186.30 Lit/seg.)

(Lts.)

(Lts.)

(Lts.)

(Lts.)

0-2

5.00%

7.550

108,720

108,720

1,087,200

978,480

2-4

15.00%

22.650

217,440

326,160

2,174,400

1,848,240

368,874
838,350

4-6

25.00%

37.750

597,960

924,120

3,261,600

2,337,480

1,777,302

6-8

85.00%

128.350

1,304,640

2,228,760

4,348,800

2,120,040

3,521,070

8-10

155.00%

234.050

1,522,080

3,750,840

5,436,000

1,685,160

5,331,906

10-12

125.00%

188.750

2,147,220

5,898,060

6,523,200

625,140

7,142,742

12-14

270.00%

407.700

2,283,120

8,181,180

7,610,400

-570,780

8,987,112

14-16

150.00%

226.500

1,386,180

9,567,360

8,697,600

-869,760

10,362,006

16-18

105.00%

158.550

1,359,000

10,926,360

9,784,800

-1,141,560

11,904,570

18-20

145.00%

218.950

1,168,740

12,095,100

10,872,000

-1,223,100

13,849,542

20-22

70.00%

105.700

652,320

12,747,420

11,959,200

-788,220

15,392,106

22-24

50.00%

75.500

298,980

13,046,400

13,046,400

16,096,320

0-2

5.00%

7.550

13,046,400

13,046,400

13,046.40

Reserv(343.68 l/s.)=

1,114,380

Reserv(343.68 l/s.)=

POR DA

1,812.00
(Lts./s.)

13,046,400
(Lts.)

(M3)

151.00

Volumen de consumo del Reservorio si se utiliza los % de variacin del 2009


DIFERENCIA

16,096,320

Vol.diseo inic.reservorio 95:

Vol.diseo ampliac.reserv.2008

1,114.0

M3

1,442.0
328.0

M3
M3

DECIDIMOS INCREMENTAR EN FORMA PARALELA UNA LINEA DE CONDUCCIN PARA INCREMENTAR EL CAUDAL DE 151 A 186.3, Qd= 35.3 Lt./seg.
POR LO QUE NECESITAMOS INCREMENTAR UN RESERVORIO DE 328 M3 PARA CUBRIR LA NECESIDAD SOLO GENERADA AL 2009
NECESITAMOS CALCULAR LA NECESIDAD PARA EL AO 2031 CONSIDERANDO UN PERIODO DE DISEO INICIAL DE 20 AOS

NDOSE EL Q=186.3 Lit/seg. Cte. como produccin


Q demanda mxima horaria = 201.83

ISTENTE ( 2000) CON PD=15 Aos

CONSUMO DETERMINADO EN BASE A LA PROYECCIN DE VARIACIONES DE CONSUMO Y EL CAUDAL DE DISEO PARA EL AO 2031, USANDO P
Q dis( 2026) =

0.00 litros/seg.

ones de consumo del ao 1999 para las zonas A y B

GASTO

Factor a =

100% demanda.2008

PROM.(2031)
@ 2 horas
% del Qprom.

DE LA LOCAL.

(Lts.)

(Lt./seg.)

(Lts./s.)

VOLUM. DE

HORAS

CONS.RESERVORIO

718,326
1,336,050
1,484,298
827,730
104,094
-619,542
-1,376,712
-1,664,406
-2,119,770
-2,977,542
-3,432,906
-3,049,920
-3,049,920

60.00%
65.00%
70.00%
85.00%
120.00%
115.00%
145.00%
125.00%
110.00%
115.00%
100.00%
90.00%
60.00%

4,917,204

4,917.2

GASTO

VOLUM.

PROM @ 2HR.

AUDAL DE 151 A 186.3, Qd= 35.3 Lt./seg.


O GENERADA AL 2009

(Lts.)

PRODUCC. PROY.

VOLUM. DE

ACUMUL.

ACUM.24Hr.2008

CONS.RESERVORIO

CONSUMO (2008)

(C / 151Lit/seg.)

100% proy.demanda.2015

(Lts.)

(Lts.)

(Lts.)
0

51.00%

0.000

2-4

55.25%

0.000

4-6

59.50%

0.000

6-8

72.25%

0.000

8-10

102.00%

0.000

10-12

97.75%

0.000

12-14

123.25%

0.000

14-16

106.25%

0.000

16-18

93.50%

0.000

18-20

97.75%

0.000

20-22

85.00%

0.000

22-24

76.50%

0.000

0-2

51.00%

0.000

0.00

Reserv(1003.28 l/s.)=

0.00
(Lts./s.)

VOLUMEN

0-2

POR DA

Vol.diseo ampliac.reserv.2008

0.85

LOCALIDAD (Consumo prom.usado en el diseo del proyecto(2000) con variaciones de consumo del ao 1999 para
las zonas A y B Pd=15 aos (2015) )
CUADRO 03.-

0
(Lts.)

0.00

(M3)

Vol.diseo inic.reservorio 2026:

0.00

Se calcula con ste dato (2031) por encontarse cercano al ao del periodo de diseo calculado de 15 aos (2012 + 15 =2027)

EL CAUDAL DE DISEO PARA EL AO 2031, USANDO PD=20 Aos (2012-2031)

RELACIN DIAMETRO - VELOCIDAD ECONMICA

diseo del proyecto(2000) con variaciones de consumo del ao 1999 para

s A y B Pd=15 aos (2015)

(Ref. : NORMAS INOS-VENEZOLANAS)


DIMETRO

M3

x 1.3 =

diseo calculado de 15 aos (2012 + 15 =2027)

0.00 M3

mm.

Pulg.

V mx. (m/seg.)

Q (m3/seg.)
3.05

75

3"

0.70

100

4"

0.75

5.89

150

6"

0.80

14.14

200

8"

0.90

28.27

250

10"

1.00

49.09

300

12"

1.10

77.75

350

14"

1.20

115.45

400

16"

1.25

157.10

450

18"

1.30

206.76

500

20"

1.40

274.90

600

24"

1.60

452.39

750

30"

1.60

729.60

You might also like